Sei sulla pagina 1di 126

1.

Using the principles of standard precautions, the nurse would wear gloves in what
nursing interventions?

A. Providing a back massage


B. Feeding a client
C. Providing hair care
D. Providing oral hygiene

2. The nurse is preparing to take vital sign in an alert client admitted to the hospital
with dehydration secondary to vomiting and diarrhea. What is the best method used
to assess the client’s temperature?

A. Oral
B. Axillary
C. Radial
D. Heat sensitive tape

3. A nurse obtained a client’s pulse and found the rate to be above normal. The
nurse document these findings as:

A. Tachypnea
B. Hyperpyrexia
C. Arrhythmia
D. Tachycardia

4. Which of the following actions should the nurse take to use a wide base support
when assisting a client to get up in a chair?

A. Bend at the waist and place arms under the client’s arms and lift
B. Face the client, bend knees and place hands on client’s forearm and lift
C. Spread his or her feet apart
D. Tighten his or her pelvic muscles
5. A client had oral surgery following a motor vehicle accident. The nurse assessing
the client finds the skin flushed and warm. Which of the following would be the best
method to take the client’s body temperature?

A. Oral
B. Axillary
C. Arterial line
D. Rectal

6. A client who is unconscious needs frequent mouth care. When performing a


mouth care, the best position of a client is:

A. Fowler’s position
B. Side lying
C. Supine
D. Trendelenburg

7. A client is hospitalized for the first time, which of the following actions ensure the
safety of the client?

A. Keep unnecessary furniture out of the way


B. Keep the lights on at all time
C. Keep side rails up at all time
D. Keep all equipment out of view

8. A walk-in client enters into the clinic with a chief complaint of


abdominal pain and diarrhea. The nurse takes the client’s vital sign hereafter. What
phrase of nursing process is being implemented here by the nurse?

A. Assessment
B. Diagnosis
C. Planning
D. Implementation

9. It is best describe as a systematic, rational method of planning and providing


nursing care for individual, families, group and community

A. Assessment
B. Nursing Process
C. Diagnosis
D. Implementation

10. Exchange of gases takes place in which of the following organ?

A. Kidney
B. Lungs
C. Liver
D. Heart

11. The chamber of the heart that receives oxygenated blood from the lungs is the:

A. Left atrium
B. Right atrium
C. Left ventricle
D. Right ventricle

12. A muscular enlarge pouch or sac that lies slightly to the left which is used for
temporary storage of food…

A. Gallbladder
B. Urinary bladder
C. Stomach
D. Lungs
13. The ability of the body to defend itself against scientific invading agent such as
baceria, toxin, viruses and foreign body

A. Hormones
B. Secretion
C. Immunity
D. Glands

14. Hormones secreted by Islets of Langerhans

A. Progesterone
B. Testosterone
C. Insulin
D. Hemoglobin

15. It is a transparent membrane that focuses the light that enters the eyes to the
retina.

A. Lens
B. Sclera
C. Cornea
D. Pupils

16. Which of the following is included in Orem’s theory?

A. Maintenance of a sufficient intake of air


B. Self perception
C. Love and belongingness
D. Physiologic needs

17. Which of the following cluster of data belong to Maslow’s hierarchy of needs
A. Love and belonging
B. Physiologic needs
C. Self actualization
D. All of the above

18. This is characterized by severe symptoms relatively of short duration.

A. Chronic Illness
B. Acute Illness
C. Pain
D. Syndrome

19. Which of the following is the nurse’s role in the health promotion

ADVERTISEMENTS

A. Health risk appraisal


B. Teach client to be effective health consumer
C. Worksite wellness
D. None of the above

20. It is described as a collection of people who share some attributes of their lives.

A. Family
B. Illness
C. Community
D. Nursing

21. Five teaspoon is equivalent to how many milliliters (ml)?

A. 30 ml
B. 25 ml
C. 12 ml
D. 22 ml

22. 1800 ml is equal to how many liters?

A. 1.8
B. 18000
C. 180
D. 2800

23. Which of the following is the abbreviation of drops?

A. Gtt.
B. Gtts.
C. Dp.
D. Dr.

24. The abbreviation for micro drop is…

A. µgtt
B. gtt
C. mdr
D. mgts

25. Which of the following is the meaning of PRN?

A. When advice
B. Immediately
C. When necessary
D. Now

Answers and Rationale


1. Answer: D. Providing oral hygiene

 Option D: Doing oral care requires the nurse to wear gloves.

2. Answer: B. Axillary

 Option B: Axilla is the most accessible body part in this situation.

3. Answer: D. Tachycardia

 Option D: Tachycardia means rapid heart rate.


 Option A: Tachypnea refers to rapid respiratory rate.
 Option B: Hyperpyrexia means increase in temperature.
 Option C: Arrhythmia means irregular heart rate.

4. Answer: B. Face the client, bend knees and place hands on client’s forearm and
lift

 Option B: This is the proper way of supporting the client to get up in a chair
that conforms to safety and proper body mechanics.

5. Answer: B. Axillary

 Option B: Taking the temperature via the axilla is the most appropriate route.
 Option A: Taking the temperature via the oral route is incorrect since the client
had oral surgery.
 Options C and D: These are unnecessary.

6. Answer: B. Side lying

 Option B: An unconscious client is best placed on his side when doing oral
care to prevent aspiration.

7. Answer: C. Keep side rails up at all time


 Option C: Keeping the side rails up at all time ensures the safety of the client.
 Options A, B, and D: Although these choices seem correct, they are not the
best answer.

8. Answer: A. Assessment

 Option A: Assessment is the first phase of the nursing process where a nurse
collects information about the client.
 Option B: Diagnosis is the formulation of the nursing diagnosis from the
information collected during the assessment.
 Option C: In Planning, the nurse sets achievable and measurable short and
long-term goals.
 Option D: Implementation is where nursing care is given.

9. Answer: B. Nursing Process

 Option B: The statement describes the Nursing Process. The Nursing Process
is the essential core of practice for the registered nurse to deliver holistic,
patient-focused care.

10. Answer: B. Lungs

 Option B: Gas exchange is the transport of oxygen from the lungs to the
bloodstream and the expulsion of carbon dioxide from the bloodstream to the
lungs. It transpires in the lungs between the alveoli and a network of tiny blood
vessels called capillaries, which are located in the walls of the alveoli.

11. Answer: A. Left atrium

 Option A: The left atrium receives oxygenated blood from the lungs and
pumps it to the left ventricle.
 Option B: The right atrium receives blood from the veins and pumps it to the
right ventricle.
 Option C: The left ventricle (the strongest chamber) pumps oxygen-rich blood
to the rest of the body, its vigorous contractions create the blood pressure.
 Option D: The right ventricle receives blood from the right atrium and pumps
it to the lungs, where it is loaded with oxygen.

12. Answer: C. Stomach

 Option C: The stomach is a muscular organ located on the left side of the
upper abdomen. It is a saclike expansion of the digestive tract of a vertebrate
that is located between the esophagus and duodenum. The major part of the
digestion of food occurs in the stomach.

13. Answer: C. Immunity

 Option C: Immunity is the ability of an organism to resist a


particular infection or toxin by the action of specific antibodies or sensitized
white blood cells.

14. Answer: C. Insulin

 Option C: The Islets of Langerhans are the regions of the pancreas that contain
its endocrine cells. Progesterone (Choice A) is produced by the ovaries.
Testosterone (Choice B) is secreted by the testicles of males and ovaries of
females. Hemoglobin (Choice D) is a protein molecule in the red blood cells
that carries oxygen from the lungs to the body’s tissues and returns carbon
dioxide.

15. Answer: C. Cornea

 Option C: The cornea is the transparent front part of the eye that covers the
iris, pupil, and anterior chamber. The cornea is like the crystal of a watch.

16. Answer: A. Maintenance of a sufficient intake of air


 Option A: Dorothea Orem’s Self-Care Theory defined Nursing as “The act of
assisting others in the provision and management of self-care to maintain or
improve human functioning at home level of effectiveness.” Choices B, C, and
D are from Abraham Maslow’s Hierarchy of Needs.

17. Answer: D. All of the above

ADVERTISEMENTS

 Option D: All of the choices are part of Maslow’s Hierarchy of Needs.

18. Answer: B. Acute Illness

 Option B: Chronic Illness (Choice A) are illnesses that are persistent or long-
term.

19. Answer: B. Teach client to be effective health consumer

 Option B: Nurses play a huge role in illness prevention and health promotion.
Nurses assume the role of ambassadors of wellness. The World Health
Organization (WHO) defines health promotion as a process of enabling people
to increase control over and to improve their health (WHO, 1986).

20. Answer: C. Community

 Option C: A community is defined by the shared attributes of the people in it,


and/or by the strength of the connections among them. When an organization is
identifying communities of interest, the shared attribute is the most useful
definition of a community.

21. Answer: B. 25 ml

 Option B: One teaspoon is equal to 5ml.


22. Answer: A. 1.8

 Option A: 1,800 ml is equal to 1.8 liters.

23. Answer: B. Gtts.

 Option B: Gtt (Choice A) is an abbreviation for drop. Dp and Dr are not


recognized abbreviation for measurement.

24. Answer: A. µgtt

 Option A: The abbreviation for micro drop is µgtt.

25. Answer: C. When necessary

 Option C: PRN comes from the Latin “pro re nata” meaning, “for an occasion
that has arisen or as circumstances require”.

1. Which of the following is the appropriate meaning of CBR?

A. Cardiac Board Room


B. Complete Bathroom
C. Complete Bed Rest
D. Complete Board Room

2. One (1) tsp is equal to how many drops?

A. 15
B. 60
C. 10
D. 30

3. 20 cc is equal to how many ml?


A. 2
B. 20
C. 2000
D. 20000

4. 1 cup is equal to how many ounces?

A. 8
B. 80
C. 800
D. 8000

5. The nurse must verify the client’s identity before administration of medication.
Which of the following is the safest way to identify the client?

A. Ask the client his name


B. Check the client’s identification band
C. State the client’s name aloud and have the client repeat it
D. Check the room number

6. The nurse prepares to administer buccal medication. The medicine should be


placed…

A. On the client’s skin


B. Between the client’s cheeks and gums
C. Under the client’s tongue
D. On the client’s conjunctiva

7. The nurse administers cleansing enema. The common position for this procedure
is…
A. Sims left lateral
B. Dorsal Recumbent
C. Supine
D. Prone

8. A client complains difficulty of swallowing when the nurse tries to administer


capsule medication. Which of the following measures should the nurse do?

A. Dissolve the capsule in a glass of water


B. Break the capsule and give the content with an applesauce
C. Check the availability of a liquid preparation
D. Crash the capsule and place it under the tongue

9. Which of the following is the appropriate route of administration for insulin?

A. Intramuscular
B. Intradermal
C. Subcutaneous
D. Intravenous

10. The nurse is ordered to administer ampicillin capsule TID p.o. The
nurse should give the medication…

A. Three times a day orally


B. Three times a day after meals
C. Two times a day by mouth
D. Two times a day before meals

11. Back Care is best described as:

A. Caring for the back by means of massage


B. Washing of the back
C. Application of cold compress at the back
D. Application of hot compress at the back

12. It refers to the preparation of the bed with a new set of linens

A. Bed bath
B. Bed making
C. Bed shampoo
D. Bed lining

13. Which of the following is the most important purpose of handwashing

A. To promote hand circulation


B. To prevent the transfer of microorganism
C. To avoid touching the client with a dirty hand
D. To provide comfort

14. What should be done in order to prevent contaminating of the environment in


bed making?

A. Avoid fanning soiled linens


B. Strip all linens at the same time
C. Finished both sides at the time
D. Embrace soiled linen

15. The most important purpose of cleansing bed bath is:

A. To cleanse, refresh and give comfort to the client who must remain in bed
B. To expose the necessary parts of the body
C. To develop skills in bed bath
D. To check the body temperature of the client in bed
16. Which of the following technique involves the sense of sight?

A. Inspection
B. Palpation
C. Percussion
D. Auscultation

17. The first techniques used examining the abdomen of a client is:

A. Palpation
B. Auscultation
C. Percussion
D. Inspection

18. A technique in physical examination that is used to assess the movement of air
through the tracheobronchial tree:

A. Palpation
B. Auscultation
C. Inspection
D. Percussion

19. An instrument used for auscultation is:

ADVERTISEMENTS

A. Percussion-hammer
B. Audiometer
C. Stethoscope
D. Sphygmomanometer

20. Resonance is best described as:


A. Sounds created by air-filled lungs
B. Short, high pitch and thudding
C. Moderately loud with musical quality
D. Drum-like

21. The best position for examining the rectum is:

A. Prone
B. Sim’s
C. Knee-chest
D. Lithotomy

22. It refers to the manner of walking

A. Gait
B. Range of motion
C. Flexion and extension
D. Hopping

23. The nurse asked the client to read the Snellen chart. Which of the following is
tested:

A. Optic
B. Olfactory
C. Oculomotor
D. Trochlear

24. Another name for knee-chest position is:

A. Genu-dorsal
B. Genu-pectoral
C. Lithotomy
D. Sim’s

25. The nurse prepares IM injection that is irritating to the subcutaneous tissue.
Which of the following is the best action in order to prevent tracking of the
medication?

A. Use a small gauge needle


B. Apply ice on the injection site
C. Administer at a 45° angle
D. Use the Z-track technique

Answers and Rationale

1. Answer: C. Complete Bed Rest

 Option C: CBR means complete bed rest. For more abbreviations, please see
this post.

2. Answer: B. 60

 Option B: One teaspoon (tsp) is equal to 60 drops (gtts).

3. Answer: B. 20

 Option B: One cubic centimeter is equal to one milliliter.

4. Answer: A. 8

 Option A: One cup is equal to 8 ounces.

5. Answer: B. Check the client’s identification band


 Option B: The identification band is the safest way to know the identity of a
patient whether he is conscious or unconscious.
 Option A: Ask the client his name only after you have checked his ID band.

6. Answer: B. Between the client’s cheeks and gums

 Option B: Buccal administration involves placing a drug between the gums


and cheek, where it also dissolves and is absorbed into the blood.

7. Answer: A. Sims left lateral

 Option A: This position provides comfort to the patient and an easy access to
the natural curvature of the rectum.

8. Answer: C. Check the availability of a liquid preparation

 Option C: The nurse should check first if the medication is available in liquid
form before doing Choice A. Placing it under the tongue is not the intended
way of administering an oral medication.

9. Answer: C. Subcutaneous

 Option C: The subcutaneous tissue of the abdomen is preferred because


absorption of the insulin is more consistent from this location than
subcutaneous tissues in other locations.

10. Answer: A. Three times a day orally

 Option A: TID is the Latin for “ter in die” which means three times a day. P.O.
means per orem or through mouth.

11. Answer: A. Caring for the back by means of massage


 Option A: Back care or massage is usually given in conjunction with the
activities of bathing the client. It can also be done on other occasions when a
client seems to have a risk of developing skin irritation due to bed rest. The
goal when performing this procedure is to enhance relaxation,
reduce muscle tension and stimulate circulation.

12. Answer: B. Bed making

 Option B: Bed making is one of the important nursing techniques to prepare


various types of bed for patients or clients to guarantee comfort and beneficial
position for a specific condition. The bed is particularly important for patients
who are sick. The nurse plays inevitable role to ensure comfort and cleanliness
for ill patient. It should be adaptable to various positions as per patient’s need
because they spend varying amount of the day in bed.

13. Answer: B. To prevent the transfer of microorganism

 Option B: Hand washing is the single most effective infection control measure.

14. Answer: A. Avoid fanning soiled linens

 Option A: Fanning soiled linens would scatter the lodged microorganisms and
dead skin cells on the linens.

15. Answer: A. To cleanse, refresh and give comfort to the client who must remain
in bed

 Option A: The nurse provides bed bath for patients who must remain in bed
and depend on someone else for their care. It is an important part of the
patient’s daily care. Not only does it remove sweat, oil, and micro-organisms
from the patient’s skin, but it also stimulates circulation and promotes a feeling
of self-worth by improving the patient’s appearance. For patients who are on
bedrest, bathing can also be a time for socialization.
16. Answer: A. Inspection

 Option A: Palpation is a method of feeling with the fingers or hands during a


physical examination. Percussion is a method of tapping on a surface to
determine the underlying structure, and is used in clinical examinations to
assess the condition of the thorax or abdomen. Auscultation (based on the
Latin verb auscultare “to listen“) is listening to the internal sounds of the body,
usually using a stethoscope.

17. Answer: D. Inspection

ADVERTISEMENTS

 Option D: For abdominal exam, auscultation is performed before palpation


because the act of palpation could change what was auscultated. Remember the
mnemonic “I-A-Per-Pal”.

18. Answer: B. Auscultation

 Option B: Auscultation of the lung is a significant part of the respiratory


examination and is necessary for diagnosing several respiratory disorders.
Auscultation assesses airflow through the trachea-bronchial tree. It is crucial to
distinguish normal respiratory sounds from abnormal ones for example
crackles, wheezes, and pleural rub in order to make a correct diagnosis.

19. Answer: C. Stethoscope

 Option C: With the invention of stethoscope by Rene Theophile Hyac in the


Laënnec in 1816; the art of auscultation not only became popular worldwide,
but also comfortable for patients and physicians.

20. Answer: A. Sounds created by air-filled lungs


 Option A: The lung is filled with air (99% of lung is air), hence, percussion of
it gives a resonance.

21. Answer: C. Knee-chest

 Option C: To assume the genu-pectoral position the person kneels so that the
weight of the body is supported by the knees and chest, with the buttocks
raised. The head is turned to one side and the arms are flexed so that the upper
part of the body can be supported in part by the elbows.

22. Answer: A. Gait

 Option A: Gait is the pattern of movement of the limbs of animals, including


humans, during locomotion over a solid substrate.

23. Answer: A. Optic

 Option A: Cranial Nerve II or the optic nerve is tested through the use of the
Snellen chart.

24. Answer: B. Genu-pectoral

 Option B: Genu-pectoral position is the position of a patient in which the


weight of the body is supported on the knees and chest.

25. Answer: D. Use the Z-track technique

 Option D: During the procedure, skin and tissue are pulled and held firmly
while a long needle is inserted into the muscle. After the medication is injected,
the skin and tissue are released. The needle track that forms during this
procedure takes the shape of the letter “Z,” which gives the procedure its name.
This zigzag track line is what prevents medication from leaking from the
muscle into surrounding tissue.
1. The most appropriate nursing order for a patient who develops dyspnea and
shortness of breath would be…

A. Maintain the patient on strict bed rest at all times


B. Maintain the patient in an orthopneic position as needed
C. Administer oxygen by Venturi mask at 24%, as needed
D. Allow a 1 hour rest period between activities

2. The nurse observes that Mr. Adams begins to have increased difficulty breathing.
She elevates the head of the bed to the high Fowler position, which decreases his
respiratory distress. The nurse documents this breathing as:

A. Tachypnea
B. Eupnea
C. Orthopnea
D. Hyperventilation

3. The physician orders a platelet count to be performed on Mrs. Smith after


breakfast. The nurse is responsible for:

A. Instructing the patient about this diagnostic test


B. Writing the order for this test
C. Giving the patient breakfast
D. All of the above

4. Mrs. Mitchell has been given a copy of her diet. The nurse discusses the foods
allowed on a 500-mg low sodium diet. These include:

A. A ham and Swiss cheese sandwich on whole wheat bread


B. Mashed potatoes and broiled chicken
C. A tossed salad with oil and vinegar and olives
D. Chicken bouillon
5. The physician orders a maintenance dose of 5,000 units of subcutaneous heparin
(an anticoagulant) daily. Nursing responsibilities for Mrs. Mitchell now include:

A. Reviewing daily activated partial thromboplastin time (APTT) and prothrombin time.
B. Reporting an APTT above 45 seconds to the physician
C. Assessing the patient for signs and symptoms of frank and occult bleeding
D. All of the above

6. The four main concepts common to nursing that appear in each of the current
conceptual models are:

A. Person, nursing, environment, medicine


B. Person, health, nursing, support systems
C. Person, health, psychology, nursing
D. Person, environment, health, nursing

7. In Maslow’s hierarchy of physiologic needs, the human need of greatest priority


is:

A. Love
B. Elimination
C. Nutrition
D. Oxygen

8. The family of an accident victim who has been declared brain-dead seems
amenable to organ donation. What should the nurse do?

A. Discourage them from making a decision until their grief has eased
B. Listen to their concerns and answer their questions honestly
C. Encourage them to sign the consent form right away
D. Tell them the body will not be available for a wake or funeral
9. A new head nurse on a unit is distressed about the poor staffing on the 11 p.m. to
7 a.m. shift. What should she do?

A. Complain to her fellow nurses


B. Wait until she knows more about the unit
C. Discuss the problem with her supervisor
D. Inform the staff that they must volunteer to rotate

10. Which of the following principles of primary nursing has proven the most
satisfying to the patient and nurse?

A. Continuity of patient care promotes efficient, cost-effective nursing care


B. Autonomy and authority for planning are best delegated to a nurse who knows the
patient well
C. Accountability is clearest when one nurse is responsible for the overall plan and its
implementation.
D. The holistic approach provides for a therapeutic relationship, continuity, and efficient
nursing care.

11. If nurse administers an injection to a patient who refuses that injection, she has
committed:

A. Assault and battery


B. Negligence
C. Malpractice
D. None of the above

12. If patient asks the nurse her opinion about a particular physicians and the nurse
replies that the physician is incompetent, the nurse could be held liable for:

A. Slander
B. Libel
C. Assault
D. Respondent superior

13. A registered nurse reaches to answer the telephone on a busy pediatric unit,
momentarily turning away from a 3 month-old infant she has been weighing. The
infant falls off the scale, suffering a skull fracture. The nurse could be charged with:

A. Defamation
B. Assault
C. Battery
D. Malpractice

14. Which of the following is an example of nursing malpractice?

A. The nurse administers penicillin to a patient with a documented history of allergy to


the drug. The patient experiences an allergic reaction and has cerebral damage resulting
from anoxia.
B. The nurse applies a hot water bottle or a heating pad to the abdomen of a patient with
abdominal cramping.
C. The nurse assists a patient out of bed with the bed locked in position; the patient slips
and fractures his right humerus.
D. The nurse administers the wrong medication to a patient and the patient vomits. This
information is documented and reported to the physician and the nursing supervisor.

15. Which of the following signs and symptoms would the nurse expect to find when
assessing an Asian patient for postoperative pain following abdominal surgery?

A. Decreased blood pressure and heart rate and shallow respirations


B. Quiet crying
C. Immobility, diaphoresis, and avoidance of deep breathing or coughing
D. Changing position every 2 hours
16. A patient is admitted to the hospital with complaints of nausea,
vomiting, diarrhea, and severe abdominal pain. Which of the following would
immediately alert the nurse that the patient has bleeding from the GI tract?

A. Complete blood count


B. Guaiac test
C. Vital signs
C. Abdominal girth

17. The correct sequence for assessing the abdomen is:

A. Tympanic percussion, measurement of abdominal girth, and inspection


B. Assessment for distention, tenderness, and discoloration around the umbilicus.
C. Percussions, palpation, and auscultation
D. Auscultation, percussion, and palpation

18. High-pitched gurgles head over the right lower quadrant are:

A. A sign of increased bowel motility


B. A sign of decreased bowel motility
C. Normal bowel sounds
D. A sign of abdominal cramping

19. A patient about to undergo abdominal inspection is best placed in which of the
following positions?

A. Prone
B. Trendelenburg
C. Supine
D. Side-lying

ADVERTISEMENTS
20. For a rectal examination, the patient can be directed to assume which of the
following positions?

A. Genupectoral
B. Sims
C. Horizontal recumbent
D. All of the above

21. During a Romberg test, the nurse asks the patient to assume which position?

A. Sitting
B. Standing
C. Genupectoral
D. Trendelenburg

22. If a patient’s blood pressure is 150/96, his pulse pressure is:

A. 54
B. 96
C. 150
D. 246

23. A patient is kept off food and fluids for 10 hours before surgery. His oral
temperature at 8 a.m. is 99.8 F (37.7 C) This temperature reading probably
indicates:

A. Infection
B. Hypothermia
C. Anxiety
D. Dehydration
24. Which of the following parameters should be checked when assessing
respirations?

A. Rate
B. Rhythm
C. Symmetry
D. All of the above

25. A 38-year old patient’s vital signs at 8 a.m. are axillary temperature 99.6 F (37.6
C); pulse rate, 88; respiratory rate, 30. Which findings should be reported?

A. Respiratory rate only


B. Temperature only
C. Pulse rate and temperature
D. Temperature and respiratory rate

Answers and Rationale

The answers and rationale below will give you a better understanding of the exam.
Counter-check your answers to those below. If you have any disputes or objects, please
direct them to the comments section.

1. Answer: B. Maintain the patient in an orthopneic position as needed

 Option B: When a patient develops dyspnea and shortness of breath, the


orthopneic position encourages maximum chest expansion and keeps
the abdominal organs from pressing against the diaphragm, thus
improving ventilation.
 Options A and C: Bed rest and oxygen by Venturi mask at 24% would
improve oxygenation of the tissues and cells but must be ordered by
a physician.
 Option D: Allowing for rest periods decreases the possibility of hypoxia.
2. Answer: C. Orthopnea

 Option C: Orthopnea is difficulty of breathing except in the upright position.


 Option A: Tachypnea is rapid respiration characterized by quick, shallow
breaths.
 Option B: Eupnea is normal respiration – quiet, rhythmic, and without effort.

3. Answer: C. Giving the patient breakfast

 Option C: A platelet count evaluates the number of platelets in the circulating


blood volume. The nurse is responsible for giving the patient breakfast at the
scheduled time.
 Options A and B: The physician is responsible for instructing the patient about
the test and for writing the order for the test.

4. Answer: B. Mashed potatoes and broiled chicken

 Option B: Mashed potatoes and broiled chicken are low in natural sodium
chloride.
 Options A, C, and D: Ham, olives, and chicken bouillon contain large
amounts of sodium and are contraindicated on a low sodium diet.

5. Answer: D. All of the above

 Option D: All of the identified nursing responsibilities are pertinent when a


patient is receiving heparin. The normal activated partial thromboplastin time is
16 to 25 seconds and the normal prothrombin time is 12 to 15 seconds; these
levels must remain within two to two and one half the normal levels.
All patients receiving anticoagulant therapy must be observed for signs and
symptoms of frank and occult bleeding (including hemorrhage, hypotension,
tachycardia, tachypnea, restlessness, pallor, cold and clammy skin, thirst
and confusion); blood pressure should be measured every 4 hours and the
patient should be instructed to report promptly any bleeding that occurs with
tooth brushing, bowel movements, urination or heavy prolonged menstruation.

6. Answer: D. Person, environment, health, nursing

 Option D: The focus concepts that have been accepted by all theorists as the
focus of nursing practice from the time of Florence Nightingale include the
person receiving nursing care, his environment, his health on the health illness
continuum, and the nursing actions necessary to meet his needs.

7. Answer: D. Oxygen

 Option D: Maslow, who defined a need as a satisfaction whose absence causes


illness, considered oxygen to be the most important physiologic need; without
it, human life could not exist.
 Options A, B, and C: According to this theory, other physiologic needs
(including food, water, elimination, shelter, rest and sleep, activity and
temperature regulation) must be met before proceeding to the next hierarchical
levels on psychosocial needs.

8. Answer: B. Listen to their concerns and answer their questions honestly

 Option B: The brain-dead patient’s family needs support and reassurance in


making a decision about organ donation.
 Option A: Because transplants are done within hours of death, decisions about
organ donation must be made as soon as possible.
 Option C: However, the family’s concerns must be addressed before members
are asked to sign a consent form.
 Option D: The body of an organ donor is available for burial.

9. Answer: C. Discuss the problem with her supervisor


 Option C: Although a new head nurse should initially spend time observing
the unit for its strengths and weakness, she should take action if a problem
threatens patient safety. In this case, the supervisor is the resource person to
approach.

10. Answer: D. The holistic approach provides for a therapeutic relationship,


continuity, and efficient nursing care.

 Option D: Studies have shown that patients and nurses both respond well to
primary nursing care units. Patients feel less anxious and isolated and more
secure because they are allowed to participate in planning their own care.
Nurses feel personal satisfaction, much of it related to positive feedback from
the patients. They also seem to gain a greater sense of achievement and esprit
de corps.

11. Answer: A. Assault and battery

 Option A: Assault is the unjustifiable attempt or threat to touch or injure


another person. Battery is the unlawful touching of another person or the
carrying out of threatened physical harm. Thus, any act that a nurse performs
on the patient against his will is considered assault and battery.

12. Answer: A. Slander

 Option A: Oral communication that injures an individual’s reputation is


considered slander.
 Option B: Written communication that does the same is considered libel.

13. Answer: D. Malpractice

 Option D: Malpractice is defined as injurious or unprofessional actions that


harm another. It involves professional misconduct, such as omission or
commission of an act that a reasonable and prudent nurse would or would not
do. In this example, the standard of care was breached; a 3-month-old infant
should never be left unattended on a scale.

14. Answer: A. The nurse administers penicillin to a patient with a documented


history of allergy to the drug. The patient experiences an allergic reaction and has
cerebral damage resulting from anoxia.

 Option A: The three elements necessary to establish a nursing malpractice


are nursing error (administering penicillin to a patient with a documented
allergy to the drug), injury (cerebral damage), and proximal cause
(administering the penicillin caused the cerebral damage).
 Option B: Applying a hot water bottle or heating pad to a patient without a
physician’s order does not include the three required components.
 Option C: Assisting a patient out of bed with the bed locked in position is the
correct nursing practice; therefore, the fracture was not the result of
malpractice.
 Option D: Administering an incorrect medication is a nursing error; however,
if such action resulted in a serious illness or chronic problem, the nurse could
be sued for malpractice.

15. Answer: C. Immobility, diaphoresis, and avoidance of deep breathing or


coughing

 Option C: An Asian patient is likely to hide his pain. Consequently, the nurse
must observe for objective signs. In an abdominal surgery patient, these might
include immobility, diaphoresis, and avoidance of deep breathing or coughing,
as well as increased heart rate, shallow respirations (stemming from pain upon
moving the diaphragm and respiratory muscles), and guarding or rigidity of the
abdominal wall. Such a patient is unlikely to display emotion, such as crying.

16. Answer: B. Guaiac test


 Option B: To assess for GI tract bleeding when frank blood is absent, the nurse
has two options: She can test for occult blood in vomitus, if present, or
in stool – through guaiac (Hemoccult) test.
 Option A: A complete blood count does not provide immediate results and
does not always immediately reflect blood loss.
 Option C: Changes in vital signs may be caused by factors other than blood
loss.
 Option D: Abdominal girth is unrelated to blood loss.

17. Answer: D. Auscultation, percussion, and palpation

ADVERTISEMENTS

 Option D: Because percussion and palpation can affect bowel motility and thus
bowel sounds, they should follow auscultation in abdominal assessment.
 Option A: Tympanic percussion, measurement of abdominal girth, and
inspection are methods of assessing the abdomen.
 Option B: Assessing for distention, tenderness and discoloration around the
umbilicus can indicate various bowel-related conditions, such
as cholecystitis, appendicitis and peritonitis.

18. Answer: C. Normal bowel sounds

 Option C: High-pitched gurgles head over the right lower quadrant are normal
bowel sounds.
 Option A: Hyperactive sounds indicate increased bowel motility.
 Option B: Two or three sounds per minute indicate decreased bowel motility.
 Option D: Abdominal cramping with hyperactive, high pitched tinkling bowel
sounds can indicate a bowel obstruction.

19. Answer: C. Supine


 Option C: The supine position (also called the dorsal position), in which the
patient lies on his back with his face upward, allows for easy access to the
abdomen.
 Option A: In the prone position, the patient lies on his abdomen with his face
turned to the side.
 Option B: In the Trendelenburg position, the head of the bed is tilted
downward to 30 to 40 degrees so that the upper body is lower than the legs.
 Option D: In the lateral position, the patient lies on his side.

20. Answer: D. All of the above

 Option D: All of these positions are appropriate for a rectal examination. In the
genupectoral (knee-chest) position, the patient kneels and rests his chest on the
table, forming a 90-degree angle between the torso and upper legs. In Sims’
position, the patient lies on his left side with the left arm behind the body and
his right leg flexed. In the horizontal recumbent position, the patient lies on his
back with legs extended and hips rotated outward.

21. Answer: B. Standing

 Option B: During a Romberg test, which evaluates for sensory or cerebellar


ataxia, the patient must stand with feet together and arms resting at the sides—
first with eyes open, then with eyes closed. The need to move the feet apart to
maintain this stance is an abnormal finding.

22. Answer: A. 54

 Option A: The pulse pressure is the difference between the systolic and
diastolic blood pressure readings – in this case, 54.

23. Answer: D. Dehydration


 Option D: A slightly elevated temperature in the immediate preoperative or
postoperative period may result from the lack of fluids before surgery rather
than from infection.
 Option C: Anxiety will not cause an elevated temperature. Hypothermia is an
abnormally low body temperature.

24 Answer D. All of the above

 Option D: The quality and efficiency of the respiratory process can be


determined by appraising the rate, rhythm, depth, ease, sound, and symmetry of
respirations.

25. Answer: D. Temperature and respiratory rate

 Option D: Under normal conditions, a healthy adult breathes in a smooth


uninterrupted pattern 12 to 20 times a minute. Thus, a respiratory rate of 30
would be abnormal. A normal adult body temperature, as measured on an oral
thermometer, ranges between 97° and 100°F (36.1° and 37.8°C); an axillary
temperature is approximately one degree lower and a rectal temperature, one
degree higher. Thus, an axillary temperature of 99.6°F (37.6°C) would be
considered abnormal. The resting pulse rate in an adult ranges from 60 to 100
beats/minute, so a rate of 88 is normal.

1. All of the following can cause tachycardia except:

A. Fever
B. Exercise
C. Sympathetic nervous system stimulation
D. Parasympathetic nervous system stimulation

2. Palpating the midclavicular line is the correct technique for assessing


A. Baseline vital signs
B. Systolic blood pressure
C. Respiratory rate
D. Apical pulse

3. The absence of which pulse may not be a significant finding when a patient is
admitted to the hospital?

A. Apical
B. Radial
C. Pedal
D. Femoral

4. Which of the following patients is at greatest risk for developing pressure ulcers?

A. An alert, chronic arthritic patient treated with steroids and aspirin


B. An 88-year old incontinent patient with gastric cancer who is confined to his bed at
home
C. An apathetic 63-year old COPD patient receiving nasal oxygen via cannula
D. A confused 78-year old patient with congestive heart failure (CHF) who requires
assistance to get out of bed.

5. The physician orders the administration of high-humidity oxygen by face mask


and placement of the patient in a high Fowler’s position. After assessing Mrs. Paul,
the nurse writes the following nursing diagnosis: Impaired gas exchange related to
increased secretions. Which of the following nursing interventions has the greatest
potential for improving this situation?

A. Encourage the patient to increase her fluid intake to 200 ml every 2 hours
B. Place a humidifier in the patient’s room.
C. Continue administering oxygen by high humidity face mask
D. Perform chest physiotherapy on a regular schedule
6. The most common deficiency seen in alcoholics is:

A. Thiamine
B. Riboflavin
C. Pyridoxine
D. Pantothenic acid

7. Which of the following statement is incorrect about a patient with dysphagia?

A. The patient will find pureed or soft foods, such as custards, easier to swallow than
water
B. Fowler’s or semi Fowler’s position reduces the risk of aspiration during swallowing
C. The patient should always feed himself
D. The nurse should perform oral hygiene before assisting with feeding.

8. To assess the kidney function of a patient with an indwelling urinary (Foley)


catheter, the nurse measures his hourly urine output. She should notify the
physician if the urine output is:

A. Less than 30 ml/hour


B. 64 ml in 2 hours
C. 90 ml in 3 hours
D. 125 ml in 4 hours

9. Certain substances increase the amount of urine produced. These include:

A. Caffeine-containing drinks, such as coffee and cola.


B. Beets
C. Urinary analgesics
D. Kaolin with pectin (Kaopectate)
10. A male patient who had surgery 2 days ago for head and neck cancer is about to
make his first attempt to ambulate outside his room. The nurse notes that he is
steady on his feet and that his vision was unaffected by the surgery. Which of the
following nursing interventions would be appropriate?

A. Encourage the patient to walk in the hall alone


B. Discourage the patient from walking in the hall for a few more days
C. Accompany the patient for his walk.
D. Consult a physical therapist before allowing the patient to ambulate

11. A patient has exacerbation of chronic obstructive pulmonary disease (COPD)


manifested by shortness of breath; orthopnea: thick, tenacious secretions; and a dry
hacking cough. An appropriate nursing diagnosis would be:

A. Ineffective airway clearance related to thick, tenacious secretions.


B. Ineffective airway clearance related to dry, hacking cough.
C. Ineffective individual coping to COPD.
D. Pain related to immobilization of affected leg.

12. Mrs. Lim begins to cry as the nurse discusses hair loss. The best response would
be:

A. “Don’t worry. It’s only temporary”


B. “Why are you crying? I didn’t get to the bad news yet”
C. “Your hair is really pretty”
D. “I know this will be difficult for you, but your hair will grow back after the
completion of chemotherapy”

13. An additional Vitamin C is required during all of the following periods except:

A. Infancy
B. Young adulthood
C. Childhood
D. Pregnancy

14. A prescribed amount of oxygen s needed for a patient with COPD to prevent:

A. Cardiac arrest related to increased partial pressure of carbon dioxide in arterial blood
(PaCO2)
B. Circulatory overload due to hypervolemia
C. Respiratory excitement
D. Inhibition of the respiratory hypoxic stimulus

15. After 1 week of hospitalization, Mr. Gray develops hypokalemia. Which of the
following is the most significant symptom of his disorder?

A. Lethargy
B. Increased pulse rate and blood pressure
C. Muscle weakness
D. Muscle irritability

16. Which of the following nursing interventions promotes patient safety?

A. Asses the patient’s ability to ambulate and transfer from a bed to a chair
B. Demonstrate the signal system to the patient
C. Check to see that the patient is wearing his identification band
D. All of the above

17. Studies have shown that about 40% of patients fall out of bed despite the use of
side rails; this has led to which of the following conclusions?

A. Side rails are ineffective


B. Side rails should not be used
C. Side rails are a deterrent that prevent a patient from falling out of bed.
D. Side rails are a reminder to a patient not to get out of bed

18. Examples of patients suffering from impaired awareness include all of the
following except:

A. A semiconscious or over fatigued patient


B. A disoriented or confused patient
C. A patient who cannot care for himself at home
D. A patient demonstrating symptoms of drugs or alcohol withdrawal

19. The most common injury among elderly persons is:

A. Atherosclerotic changes in the blood vessels


B. Increased incidence of gallbladder disease
C. Urinary Tract Infection
D. Hip fracture

ADVERTISEMENTS

20. The most common psychogenic disorder among elderly person is:

A. Depression
B. Sleep disturbances (such as bizarre dreams)
C. Inability to concentrate
D. Decreased appetite

21. Which of the following vascular system changes results from aging?

A. Increased peripheral resistance of the blood vessels


B. Decreased blood flow
C. Increased workload of the left ventricle
D. All of the above
22. Which of the following is the most common cause of dementia among elderly
persons?

A. Parkinson’s disease
B. Multiple sclerosis
C. Amyotrophic lateral sclerosis (Lou Gehrig’s disease)
D. Alzheimer’s disease

23. The nurse’s most important legal responsibility after a patient’s death in a
hospital is:

A. Obtaining a consent of an autopsy


B. Notifying the coroner or medical examiner
C. Labeling the corpse appropriately
D. Ensuring that the attending physician issues the death certification

24. Before rigor mortis occurs, the nurse is responsible for:

A. Providing a complete bath and dressing change


B. Placing one pillow under the body’s head and shoulders
C. Removing the body’s clothing and wrapping the body in a shroud
D. Allowing the body to relax normally

25. When a patient in the terminal stages of lung cancer begins to exhibit loss of
consciousness, a major nursing priority is to:

A. Protect the patient from injury


B. Insert an airway
C. Elevate the head of the bed
D. Withdraw all pain medications

Answers and Rationale


The answers and rationale below will give you a better understanding of the exam.
Counter-check your answers to those below. If you have any disputes or objects, please
direct them to the comments section.

1. Answer: D. Parasympathetic nervous system stimulation

 Option D: Parasympathetic nervous system stimulation of the heart decreases


the heart rate as well as the force of contraction, rate of impulse conduction and
blood flow through the coronary vessels. Fever, exercise, and sympathetic
stimulation all increase the heart rate.

2. Answer: D. Apical pulse

 Option D: The apical pulse (the pulse at the apex of the heart) is located on the
midclavicular line at the fourth, fifth, or sixth intercostal space. Base line vital
signs include pulse rate, temperature, respiratory rate, and blood pressure.
Blood pressure is typically assessed at the antecubital fossa, and respiratory rate
is assessed best by observing chest movement with
each inspiration and expiration.

3. Answer: C. Pedal

 Option C: Because the pedal pulse cannot be detected in 10% to 20% of the
population, its absence is not necessarily a significant finding. However, the
presence or absence of the pedal pulse should be documented upon admission
so that changes can be identified during the hospital stay.
 Options A, B, and D: Absence of the apical, radial, or femoral pulse is
abnormal and should be investigated.

4. Answer: B. An 88-year old incontinent patient with gastric cancer who is confined
to his bed at home
 Option B: Pressure ulcers are most likely to develop in patients with impaired
mental status, mobility, activity level, nutrition, circulation and bladder or
bowel control. Age is also a factor. Thus, the 88-year old incontinent patient
who has impaired nutrition (from gastric cancer) and is confined to bed is at
greater risk.

5. Answer: A. Encourage the patient to increase her fluid intake to 200 ml every 2
hours

 Option A: Adequate hydration thins and loosens pulmonary secretions and also
helps to replace fluids lost from elevated temperature, diaphoresis, dehydration,
and dyspnea.
 Options B, C, and D: High-humidity air and chest physiotherapy help liquefy
and mobilize secretions.

6. Answer: A. Thiamine

 Option A: Chronic alcoholism commonly results in thiamine deficiency and


other symptoms of malnutrition.

7. Answer: C. The patient should always feed himself

 Option C: A patient with dysphagia (difficulty swallowing) requires assistance


with feeding. Feeding himself is a long-range expected outcome.
 Options A, B, and D: Soft foods, Fowler’s or semi-Fowler’s position, and oral
hygiene before eating should be part of the feeding regimen.

8. Answer: A. Less than 30 ml/hour

 Option A: A urine output of less than 30ml/hour indicates hypovolemia or


oliguria, which is related to kidney function and inadequate fluid intake.

9. Answer: A. Caffeine-containing drinks, such as coffee and cola.


 Option A: Fluids containing caffeine have a diuretic effect.
 Options B and C: Beets and urinary analgesics, such as
Pyridium (Phenazopyridine), can color urine red.
 Option D: Kaopectate is an antidiarrheal medication.

10. Answer: C. Accompany the patient for his walk.

 Option C: Accompanying him will offer moral support, enabling him to face
the rest of the world.
 Option A: A hospitalized surgical patient leaving his room for the first time
fears rejection and others staring at him, so he should not walk alone.
 Option B: Patients should begin ambulation as soon as possible after surgery to
decrease complications and to regain strength and confidence.
 Option D: Waiting to consult a physical therapist is unnecessary.

11. Answer: A. Ineffective airway clearance related to thick, tenacious secretions.

 Option A: Thick, tenacious secretions, a dry, hacking cough, orthopnea, and


shortness of breath are signs of ineffective airway clearance.
 Option B: Ineffective airway clearance related to dry, hacking cough is
incorrect because the cough is not the reason for the ineffective airway
clearance.
 Option C: Ineffective individual coping related to COPD is wrong because the
etiology for a nursing diagnosis should not be a medical diagnosis (COPD) and
because no data indicate that the patient is coping ineffectively.
 Option D: Pain related to immobilization of affected leg would be an
appropriate nursing diagnosis for a patient with a leg fracture.

12. Answer: D. “I know this will be difficult for you, but your hair will grow back
after the completion of chemotherapy”

 Option D: “I know this will be difficult” acknowledges the problem and


suggests a resolution to it.
 Option A: “Don’t worry..” offers some relief but doesn’t recognize the
patient’s feelings.
 Option B: “..I didn’t get to the bad news yet” would be inappropriate at any
time.
 Option C: “Your hair is really pretty” offers no consolation or alternatives to
the patient.

13. Answer: B. Young adulthood

 Option B: Additional Vitamin C is needed in growth periods, such as infancy


and childhood, and during pregnancy to supply demands for fetal growth and
maternal tissues. Other conditions requiring extra vitamin C include wound
healing, fever, infection and stress.

14. Answer: D. Inhibition of the respiratory hypoxic stimulus

 Option D: Delivery of more than 2 liters of oxygen per minute to a patient with
chronic obstructive pulmonary disease (COPD), who is usually in a state of
compensated respiratory acidosis (retaining carbon dioxide (CO2)), can inhibit
the hypoxic stimulus for respiration.
 Option A: An increased partial pressure of carbon dioxide in arterial blood
(PACO2) would not initially result in cardiac arrest.
 Options B and C: Circulatory overload and respiratory excitement have no
relevance to the question.

15. Answer: C. Muscle weakness

 Option C: Presenting symptoms of hypokalemia ( a serum potassium level


below 3.5 mEq/liter) include muscle weakness, chronic fatigue, and cardiac
dysrhythmias. The combined effects of inadequate food intake and
prolonged diarrhea can deplete the potassium stores of a patient with GI
problems.
16. Answer: D. All of the above

 Option D: Assisting a patient with ambulation and transfer from a bed to a


chair allows the nurse to evaluate the patient’s ability to carry out these
functions safely. Demonstrating the signal system and providing an opportunity
for a return demonstration ensures that the patient knows how to operate the
equipment and encourages him to call for assistance when needed. Checking
the patient’s identification band verifies the patient’s identity and prevents
identification mistakes in drug administration.

17. Answer: D. Side rails are a reminder to a patient not to get out of bed

ADVERTISEMENTS

 Option D: Since about 40% of patients fall out of bed despite the use of side
rails, side rails cannot be said to prevent falls; however, they do serve as a
reminder that the patient should not get out of bed.
 Options A, B, and C: The other answers are incorrect interpretations of the
statistical data.

18. Answer: C. A patient who cannot care for himself at home

 Option C: A patient who cannot care for himself at home does not necessarily
have impaired awareness; he may simply have some degree of immobility.

19. Answer: D. Hip fracture

 Option D: Hip fracture, the most common injury among elderly persons,
usually results from osteoporosis.
 Options A, B, and C: The other answers are diseases that can occur in the
elderly from physiologic changes.

20. Answer: A. Depression


 Option A: Depression typically begins before the onset of old age and usually
is caused by psychosocial, genetic, or biochemical factors.
 Options B, C, and D: Sleep disturbances, inability to concentrate and
decreased appetite are symptoms of depression, the most common psychogenic
disorder among elderly persons. Other symptoms include diminished memory,
apathy, disinterest in appearance, withdrawal, and irritability.

21. Answer: D. All of the above

 Option D: Aging decreases the elasticity of the blood vessels, which leads to
increased peripheral resistance and decreased blood flow. These changes, in
turn, increase the workload of the left ventricle.

22. Answer: D. Alzheimer’s disease

 Option D: Alzheimer;s disease, sometimes known as senile dementia of the


Alzheimer’s type or primary degenerative dementia, is an insidious;
progressive, irreversible, and degenerative disease of the brain whose etiology
is still unknown.
 Option A: Parkinson’s disease is a neurologic disorder caused by lesions in the
extrapyramidal system and manifested by tremors, muscle rigidity,
hypokinesia, dysphagia, and dysphonia.
 Option B: Multiple sclerosis, a progressive, degenerative disease involving
demyelination of the nerve fibers, usually begins in young adulthood and is
marked by periods of remission and exacerbation.
 Option C: Amyotrophic lateral sclerosis, a disease marked by progressive
degeneration of the neurons, eventually results in atrophy of all the muscles;
including those necessary for respiration.

23. Answer: C. Labeling the corpse appropriately

 Option C: The nurse is legally responsible for labeling the corpse when death
occurs in the hospital.
 Options A and B: She may be involved in obtaining consent for an autopsy or
notifying the coroner or medical examiner of a patient’s death; however, she is
not legally responsible for performing these functions.
 Option D: The attending physician may need information from the nurse to
complete the death certificate, but he is responsible for issuing it.

24. Answer: B. Placing one pillow under the body’s head and shoulders

 Option B: The nurse must place a pillow under the deceased person’s head and
shoulders to prevent blood from settling in the face and discoloring it.
 Option A: She is required to bathe only soiled areas of the body since the
mortician will wash the entire body.
 Option C: Before wrapping the body in a shroud, the nurse places a clean
gown on the body and closes the eyes and mouth.

25. Answer: A. Protect the patient from injury

 Option A: Ensuring the patient’s safety is the most essential action at this time.
 Options B, C, and D: The other nursing actions may be necessary but are not a
major priority.

1. Which element in the circular chain of infection can be eliminated by


preserving skin integrity?

A. Host
B. Reservoir
C. Mode of transmission
D. Portal of entry

2. Which of the following will probably result in a break in sterile technique for
respiratory isolation?
A. Opening the patient’s window to the outside environment
B. Turning on the patient’s room ventilator
C. Opening the door of the patient’s room leading into the hospital corridor
D. Failing to wear gloves when administering a bed bath

3. Which of the following patients is at greater risk for contracting an infection?

A. A patient with leukopenia


B. A patient receiving broad-spectrum antibiotics
C. A postoperative patient who has undergone orthopedic surgery
D. A newly diagnosed diabetic patient

4. Effective hand washing requires the use of:

A. Soap or detergent to promote emulsification


B. Hot water to destroy bacteria
C. A disinfectant to increase surface tension
D. All of the above

5. After routine patient contact, hand washing should last at least:

A. 30 seconds
B. 1 minute
C. 2 minutes
D. 3 minutes

6. Which of the following procedures always requires surgical asepsis?

A. Vaginal instillation of conjugated estrogen


B. Urinary catheterization
C. Nasogastric tube insertion
D. Colostomy irrigation
7. Sterile technique is used whenever:

A. Strict isolation is required


B. Terminal disinfection is performed
C. Invasive procedures are performed
D. Protective isolation is necessary

8. Which of the following constitutes a break in sterile technique while preparing a


sterile field for a dressing change?

A. Using sterile forceps, rather than sterile gloves, to handle a sterile item
B. Touching the outside wrapper of sterilized material without sterile gloves
C. Placing a sterile object on the edge of the sterile field
D. Pouring out a small amount of solution (15 to 30 ml) before pouring the solution into a
sterile container

9. A natural body defense that plays an active role in preventing infection is:

A. Yawning
B. Body hair
C. Hiccupping
D. Rapid eye movements

10. All of the following statement are true about donning sterile gloves except:

A. The first glove should be picked up by grasping the inside of the cuff.
B. The second glove should be picked up by inserting the gloved fingers under the cuff
outside the glove.
C. The gloves should be adjusted by sliding the gloved fingers under the sterile cuff and
pulling the glove over the wrist
D. The inside of the glove is considered sterile
11. When removing a contaminated gown, the nurse should be careful that the first
thing she touches is the:

A. Waist tie and neck tie at the back of the gown


B. Waist tie in front of the gown
C. Cuffs of the gown
D. Inside of the gown

12. Which of the following nursing interventions is considered the most effective
form or universal precautions?

A. Cap all used needles before removing them from their syringes
B. Discard all used uncapped needles and syringes in an impenetrable protective
container
C. Wear gloves when administering IM injections
D. Follow enteric precautions

13. All of the following measures are recommended to prevent pressure


ulcers except:

A. Massaging the reddened area with lotion


B. Using a water or air mattress
C. Adhering to a schedule for positioning and turning
D. Providing meticulous skin care

14. Which of the following blood tests should be performed before a blood
transfusion?

A. Prothrombin and coagulation time


B. Blood typing and cross-matching
C. Bleeding and clotting time
D. Complete blood count (CBC) and electrolyte levels.
15. The primary purpose of a platelet count is to evaluate the:

A. Potential for clot formation


B. Potential for bleeding
C. Presence of an antigen-antibody response
D. Presence of cardiac enzymes

16. Which of the following white blood cell (WBC) counts clearly indicates
leukocytosis?

A. 4,500/mm³
B. 7,000/mm³
C. 10,000/mm³
D. 25,000/mm³

17. After 5 days of diuretic therapy with 20mg of furosemide (Lasix) daily, a patient
begins to exhibit fatigue, muscle cramping and muscle weakness. These symptoms
probably indicate that the patient is experiencing:

A. Hypokalemia
B. Hyperkalemia
C. Anorexia
D. Dysphagia

18. Which of the following statements about chest X-ray is false?

A. No contradictions exist for this test


B. Before the procedure, the patient should remove all jewelry, metallic objects, and
buttons above the waist
C. A signed consent is not required
D. Eating, drinking, and medications are allowed before this test
19. The most appropriate time for the nurse to obtain a sputum specimen for
culture is:

A. Early in the morning


B. After the patient eats a light breakfast
C. After aerosol therapy
D. After chest physiotherapy

20. A patient with no known allergies is to receive penicillin every 6 hours. When
administering the medication, the nurse observes a fine rash on the patient’s skin.
The most appropriate nursing action would be to:

ADVERTISEMENTS

A. Withhold the moderation and notify the physician


B. Administer the medication and notify the physician
C. Administer the medication with an antihistamine
D. Apply cornstarch soaks to the rash

21. All of the following nursing interventions are correct when using the Z-track
method of drug injection except:

A. Prepare the injection site with alcohol


B. Use a needle that’s a least 1” long
C. Aspirate for blood before injection
D. Rub the site vigorously after the injection to promote absorption

22. The correct method for determining the vastus lateralis site for I.M. injection is
to:

A. Locate the upper aspect of the upper outer quadrant of the buttock about 5 to 8 cm
below the iliac crest
B. Palpate the lower edge of the acromion process and the midpoint lateral aspect of the
arm
C. Palpate a 1” circular area anterior to the umbilicus
D. Divide the area between the greater femoral trochanter and the lateral femoral condyle
into thirds, and select the middle third on the anterior of the thigh

23. The mid-deltoid injection site is seldom used for I.M. injections because it:

A. Can accommodate only 1 ml or less of medication


B. Bruises too easily
C. Can be used only when the patient is lying down
D. Does not readily parenteral medication

24. The appropriate needle size for insulin injection is:

A. 18G, 1 ½” long
B. 22G, 1” long
C. 22G, 1 ½” long
D. 25G, 5/8” long

25. The appropriate needle gauge for intradermal injection is:

A. 20G
B. 22G
C. 25G
D. 26G

Answers and Rationale

Gauge your performance by counter checking your answers to the answers below. Learn
more about the question by reading the rationale. If you have any disputes or questions,
please direct them to the comments section.
1. Answer: D. Portal of entry

Option D: In the circular chain of infection, pathogens must be able to leave their
reservoir and be transmitted to a susceptible host through a portal of entry, such as broken
skin.

2. Answer: C. Opening the door of the patient’s room leading into the hospital
corridor

 Option C: Respiratory isolation, like strict isolation, requires that the door to
the door patient’s room remain closed.
 Options A and B: However, the patient’s room should be well ventilated, so
opening the window or turning on the ventricular is desirable.
 Option D: The nurse does not need to wear gloves for respiratory isolation, but
good hand washing is important for all types of isolation.

3. Answer: A. A patient with leukopenia

 Option A: Leukopenia is a decreased number of leukocytes (white blood cells),


which are important in resisting infection.

Options B, C, and D: None of the other situations would put the patient at risk for
contracting an infection; taking broad-spectrum antibiotics might actually reduce the
infection risk.

4. Answer: A. Soap or detergent to promote emulsification

 Option A: Soaps and detergents are used to help remove bacteria because of
their ability to lower the surface tension of water and act as emulsifying agents.
 Option B: Hot water may lead to skin irritation or burns.

5. Answer: A. 30 seconds
 Option A: Depending on the degree of exposure to pathogens, hand
washing may last from 10 seconds to 4 minutes. After routine patient
contact, hand washing for 30 seconds effectively minimizes the risk of
pathogen transmission.

6. Answer: B. Urinary catheterization

 Option B: The urinary system is normally free of microorganisms except at the


urinary meatus. Any procedure that involves entering this system must use
surgically aseptic measures to maintain a bacteria-free state.

7. Answer: C. Invasive procedures are performed

 Option C: All invasive procedures, including surgery, catheter insertion, and


administration of parenteral therapy, require sterile technique to maintain a
sterile environment. All equipment must be sterile, and the nurse and the
physician must wear sterile gloves and maintain surgical asepsis. In the
operating room, the nurse and physician are required to wear sterile gowns,
gloves, masks, hair covers, and shoe covers for all invasive procedures.
 Option A: Strict isolation requires the use of clean gloves, masks, gowns and
equipment to prevent the transmission of highly communicable diseases by
contact or by airborne routes.
 Option B: Terminal disinfection is the disinfection of all contaminated supplies
and equipment after a patient has been discharged to prepare them for reuse by
another patient.
 Option D: The purpose of protective (reverse)isolation is to prevent a person
with seriously impaired resistance from coming into contact who potentially
pathogenic organisms.

8. Answer: C. Placing a sterile object on the edge of the sterile field


 Option C: The edges of a sterile field are considered contaminated. When
sterile items are allowed to come in contact with the edges of the field, the
sterile items also become contaminated.

9. Answer: B. Body hair

 Option B: Hair on or within body areas, such as the nose, traps and holds
particles that contain microorganisms.
 Options A and C: Yawning and hiccupping do not prevent microorganisms
from entering or leaving the body.
 Option D: Rapid eye movement marks the stage of sleep during which
dreaming occurs.

10. Answer: D. The inside of the glove is considered sterile

 Option D: The inside of the glove is always considered to be clean, but not
sterile.

11. Answer: A. Waist tie and neck tie at the back of the gown

 Option A: The back of the gown is considered clean, the front is contaminated.
So, after removing gloves and washing hands, the nurse should untie the back
of the gown; slowly move backward away from the gown, holding the inside of
the gown and keeping the edges off the floor; turn and fold the gown inside out;
discard it in a contaminated linen container; then wash her hands again.

12. Answer: B. Discard all used uncapped needles and syringes in an impenetrable
protective container

 Option B: According to the Centers for Disease Control (CDC), blood-to-


blood contact occurs most commonly when a health care worker attempts to
cap a used needle.
 Option A: Therefore, used needles should never be recapped; instead they
should be inserted in a specially designed puncture resistant, labeled container.
 Option C: Wearing gloves is not always necessary when administering an I.M.
injection.
 Option D: Enteric precautions prevent the transfer of pathogens via feces.

13. Answer: A. Massaging the reddened area with lotion

 Option A: Nurses and other healthcare professionals previously believed that


massaging a reddened area with lotion would promote venous return and reduce
edema to the area. However, research has shown that massage only increases
the likelihood of cellular ischemia and necrosis to the area.

14. Answer: B. Blood typing and cross-matching

 Option B: Before a blood transfusion is performed, the blood of the donor and
recipient must be checked for compatibility. This is done by blood typing (a
test that determines a person’s blood type) and cross-matching (a procedure that
determines the compatibility of the donor’s and recipient’s blood after the
blood types has been matched). If the blood specimens are incompatible,
hemolysis and antigen-antibody reactions will occur.

15. Answer: A. Potential for clot formation

 Option A: Platelets are disk-shaped cells that are essential for blood
coagulation. A platelet count determines the number of thrombocytes in blood
available for promoting hemostasis and assisting with blood coagulation after
injury.
 Option B: It also is used to evaluate the patient’s potential for bleeding;
however, this is not its primary purpose. The normal count ranges from
150,000 to 350,000/mm3. A count of 100,000/mm3 or less indicates a potential
for bleeding; count of less than 20,000/mm3 is associated with
spontaneous bleeding.
16. Answer: D. 25,000/mm³

 Option D: Leukocytosis is any transient increase in the number of white blood


cells (leukocytes) in the blood. Normal WBC counts range from 5,000 to
100,000/mm3. Thus, a count of 25,000/mm3 indicates leukocytosis.

17. Answer: A. Hypokalemia

ADVERTISEMENTS

 Option A: Fatigue, muscle cramping, and muscle weaknesses are symptoms


of hypokalemia (an inadequate potassium level), which is a potential side effect
of diuretic therapy. The physician usually orders supplemental potassium to
prevent hypokalemia in patients receiving diuretics.
 Option C: Anorexia is another symptom of hypokalemia.
 Option D: Dysphagia means difficulty swallowing.

18. Answer: A. No contradictions exist for this test

 Option A: Pregnancy or suspected pregnancy is the only contraindication for a


chest X-ray. However, if a chest X-ray is necessary, the patient can wear a lead
apron to protect the pelvic region from radiation.
 Option B: Jewelry, metallic objects, and buttons would interfere with the X-
ray and thus should not be worn above the waist.
 Option C: A signed consent is not required because a chest X-ray is not an
invasive examination.
 Option D: Eating, drinking and medications are allowed because the X-ray is
of the chest, not the abdominal region.

19. Answer: A. Early in the morning


 Option A: Obtaining a sputum specimen early in this morning ensures an
adequate supply of bacteria for culturing and decreases the risk of
contamination from food or medication.

20. Answer: A. Withhold the moderation and notify the physician

 Option A: Initial sensitivity to penicillin is commonly manifested by a skin


rash, even in individuals who have not been allergic to it previously. Because of
the danger of anaphylactic shock, the nurse should withhold the drug and notify
the physician, who may choose to substitute another drug.
 Option C: Administering an antihistamine is a dependent nursing intervention
that requires a written physician’s order.
 Option D: Although applying cornstarch to the rash may relieve discomfort, it
is not the nurse’s top priority in such a potentially life-threatening situation.

21. Answer: D. Rub the site vigorously after the injection to promote absorption

 Option D: The Z-track method is an I.M. injection technique in which the


patient’s skin is pulled in such a way that the needle track is sealed off after the
injection. This procedure seals medication deep into the muscle, thereby
minimizing skin staining and irritation. Rubbing the injection site is
contraindicated because it may cause the medication to extravasate into the
skin.

22. Answer: D. Divide the area between the greater femoral trochanter and the
lateral femoral condyle into thirds, and select the middle third on the anterior of the
thigh

 Option D: The vastus lateralis, a long, thick muscle that extends the full length
of the thigh, is viewed by many clinicians as the site of choice for I.M.
injections because it has relatively few major nerves and blood vessels. The
middle third of the muscle is recommended as the injection site. The patient can
be in a supine or sitting position for an injection into this site.
23. Answer: A. Can accommodate only 1 ml or less of medication

 Option A: The mid-deltoid injection site can accommodate only 1 ml or less of


medication because of its size and location (on the deltoid muscle of the arm,
close to the brachial artery and radial nerve).

24. Answer: D. 25G, 5/8” long

 Option D: A 25G, 5/8” needle is the recommended size for insulin injection
because insulin is administered by the subcutaneous route.
 Option A: An 18G, 1 ½” needle is usually used for I.M. injections in children,
typically in the vastus lateralis.
 Option C: A 22G, 1 ½” needle is usually used for adult I.M. injections, which
are typically administered in the vastus lateralis or ventrogluteal site.

25. Answer: D. 26G

 Option D: Because an intradermal injection does not penetrate deeply into the
skin, a small-bore 26G-27G needle is recommended. This type of injection is
used primarily to administer antigens to evaluate reactions for allergy or
sensitivity studies.
Options A, B, and C: A 20G needle is usually used for I.M. injections of oil-
based medications; a 22G-25G needle for I.M. injections; and a 25G needle, for
subcutaneous insulin injections.

1. Parenteral penicillin can be administered as an:

A. IM injection or an IV solution
B. IV or an intradermal injection
C. Intradermal or subcutaneous injection
D. IM or a subcutaneous injection
2. The physician orders gr 10 of aspirin for a patient. The equivalent dose in
milligrams is:

A. 0.6 mg
B. 10 mg
C. 60 mg
D. 600 mg

3. The physician orders an IV solution of dextrose 5% in water at 100ml/hour. What


would the flow rate be if the drop factor is 15 gtt = 1 ml?

A. 5 gtt/minute
B. 13 gtt/minute
C. 25 gtt/minute
D. 50 gtt/minute

4. Which of the following is a sign or symptom of a hemolytic reaction


to blood transfusion?

A. Hemoglobinuria
B. Chest pain
C. Urticaria
D. Distended neck veins

5. Which of the following conditions may require fluid restriction?

A. Fever
B. Chronic Obstructive Pulmonary Disease
C. Renal Failure
D. Dehydration

6. All of the following are common signs and symptoms of phlebitis except:
A. Pain or discomfort at the IV insertion site
B. Edema and warmth at the IV insertion site
C. A red streak exiting the IV insertion site
D. Frank bleeding at the insertion site

7. The best way of determining whether a patient has learned to instill


ear medication properly is for the nurse to:

A. Ask the patient if he/she has used ear drops before


B. Have the patient repeat the nurse’s instructions using her own words
C. Demonstrate the procedure to the patient and encourage to ask questions
D. Ask the patient to demonstrate the procedure

8. Which of the following types of medications can be administered via gastrostomy


tube?

A. Any oral medications


B. Capsules whole contents are dissolved in water
C. Enteric-coated tablets that are thoroughly dissolved in water
D. Most tablets designed for oral use, except for extended-duration compounds

9. A patient who develops hives after receiving an antibiotic is exhibiting drug:

A. Tolerance
B. Idiosyncrasy
C. Synergism
D. Allergy

10. A patient has returned to his room after femoral arteriography. All of the
following are appropriate nursing interventions except:
A. Assess femoral, popliteal, and pedal pulses every 15 minutes for 2 hours
B. Check the pressure dressing for sanguineous drainage
C. Assess vital signs every 15 minutes for 2 hours
D. Order a hemoglobin and hematocrit count 1 hour after the arteriography

11. The nurse explains to a patient that a cough:

A. Is a protective response to clear the respiratory tract of irritants


B. Is primarily a voluntary action
C. Is induced by the administration of an antitussive drug
D. Can be inhibited by “splinting” the abdomen

12. An infected patient has chills and begins shivering. The best nursing
intervention is to:

A. Apply iced alcohol sponges


B. Provide increased cool liquids
C. Provide additional bedclothes
D. Provide increased ventilation

13. A clinical nurse specialist is a nurse who has:

A. Been certified by the National League for Nursing


B. Received credentials from the American Nurses’ Association
C. Graduated from an associate degree program and is a registered professional nurse
D. Completed a master’s degree in the prescribed clinical area and is a registered
professional nurse.

14. The purpose of increasing urine acidity through dietary means is to:

A. Decrease burning sensations


B. Change the urine’s color
C. Change the urine’s concentration
D. Inhibit the growth of microorganisms

15. Clay-colored stools indicate:

A. Upper GI bleeding
B. Impending constipation
C. An effect of medication
D. Bile obstruction

16. In which step of the nursing process would the nurse ask a patient if the
medication she administered relieved his pain?

A. Assessment
B. Analysis
C. Planning
D. Evaluation

17. All of the following are good sources of vitamin A except:

A. White potatoes
B. Carrots
C. Apricots
D. Egg yolks

18. Which of the following is a primary nursing intervention necessary for all
patients with a Foley Catheter in place?

A. Maintain the drainage tubing and collection bag level with the patient’s bladder
B. Irrigate the patient with 1% Neosporin solution three times a daily
C. Clamp the catheter for 1 hour every 4 hours to maintain the bladder’s elasticity
D. Maintain the drainage tubing and collection bag below bladder level to facilitate
drainage by gravity

19. The ELISA test is used to:

A. Screen blood donors for antibodies to human immunodeficiency virus (HIV)


B. Test blood to be used for transfusion for HIV antibodies
C. Aid in diagnosing a patient with AIDS
D. All of the above

ADVERTISEMENTS

20. The two blood vessels most commonly used for TPN infusion are the:

A. Subclavian and jugular veins


B. Brachial and subclavian veins
C. Femoral and subclavian veins
D. Brachial and femoral veins

21. Effective skin disinfection before a surgical procedure includes which of the
following methods?

A. Shaving the site on the day before surgery


B. Applying a topical antiseptic to the skin on the evening before surgery
C. Having the patient take a tub bath on the morning of surgery
D. Having the patient shower with an antiseptic soap on the evening before and the
morning of surgery

22. When transferring a patient from a bed to a chair, the nurse should use which
muscles to avoid back injury?

A. Abdominal muscles
B. Back muscles
C. Leg muscles
D. Upper arm muscles

23. Thrombophlebitis typically develops in patients with which of the following


conditions?

A. Increases partial thromboplastin time


B. Acute pulsus paradoxus
C. An impaired or traumatized blood vessel wall
D. Chronic Obstructive Pulmonary Disease (COPD)

24. In a recumbent, immobilized patient, lung ventilation can become altered,


leading to such respiratory complications as:

A. Respiratory acidosis, atelectasis, and hypostatic pneumonia


B. Apneustic breathing, atypical pneumonia and respiratory alkalosis
C. Cheyne-Stokes respirations and spontaneous pneumothorax
D. Kussmaul’s respirations and hypoventilation

25. Immobility impairs bladder elimination, resulting in such disorders as

A. Increased urine acidity and relaxation of the perineal muscles, causing incontinence
B. Urine retention, bladder distention, and infection
C. Diuresis, natriuresis, and decreased urine specific gravity
D. Decreased calcium and phosphate levels in the urine

Answers and Rationale

Gauge your performance by counter checking your answers to the answers below. Learn
more about the question by reading the rationale. If you have any disputes or questions,
please direct them to the comments section.
1. Answer: A. IM injection or an IV solution

 Option A: Parenteral penicillin can be administered I.M. or added to a solution


and given I.V. It cannot be administered subcutaneously or intradermally.

2. Answer: D. 600 mg

 Option D: gr 10 x 60 mg/gr 1 = 600 mg

3. Answer: C. 25 gtt/minute

 Option C: 100ml/60 min X 15 gtt/ 1 ml = 25 gtt/minute

4. Answer: A. Hemoglobinuria

 Option A: Hemoglobinuria, the abnormal presence of hemoglobin in the urine,


indicates a hemolytic reaction (incompatibility of the donor’s and recipient’s
blood). In this reaction, antibodies in the recipient’s plasma combine rapidly
with donor RBC’s; the cells are hemolyzed in either circulatory or
reticuloendothelial system. Hemolysis occurs more rapidly in ABO
incompatibilities than in Rh incompatibilities.
 Options B and C: Chest pain and urticaria may be symptoms of
impending anaphylaxis.
 Option D: Distended neck veins are an indication of hypervolemia.

5. Answer: C. Renal Failure

 Option C: In renal failure, the kidney loses their ability to effectively eliminate
wastes and fluids. Because of this, limiting the patient’s intake of oral and I.V.
fluids may be necessary.
 Options A, B, and D: Fever, chronic obstructive pulmonary disease,
and dehydration are conditions for which fluids should be encouraged.
6. Answer: D. Frank bleeding at the insertion site

 Option D: Phlebitis, the inflammation of a vein, can be caused by chemical


irritants (I.V. solutions or medications), mechanical irritants (the needle or
catheter used during venipuncture or cannulation), or a localized allergic
reaction to the needle or catheter.
 Options A, B, and C: Signs and symptoms of phlebitis include pain or
discomfort, edema and heat at the I.V. insertion site, and a red streak going up
the arm or leg from the I.V. insertion site.

7. Answer: D. Ask the patient to demonstrate the procedure

 Option D: Return demonstration provides the most certain evidence for


evaluating the effectiveness of patient teaching.

8. Answer: D. Most tablets designed for oral use, except for extended-duration
compounds

 Option D: Most tablets designed for oral use, except for extended-duration
compounds can be administered via gastrostomy tube.
 Options A, B, and C: Capsules, enteric-coated tablets, and most extended
duration or sustained release products should not be dissolved for use in a
gastrostomy tube. They are pharmaceutically manufactured in these forms for
valid reasons, and altering them destroys their purpose. The nurse should seek
an alternate physician’s order when an ordered medication is inappropriate
for delivery by tube.

9. Answer: D. Allergy

 Option D: A drug-allergy is an adverse reaction resulting from an


immunologic response following a previous sensitizing exposure to the drug.
The reaction can range from a rash or hives to anaphylactic shock.
 Option A: Tolerance to a drug means that the patient experiences a decreasing
physiologic response to repeated administration of the drug in the same dosage.
 Option B: Idiosyncrasy is an individual’s unique hypersensitivity to a drug,
food, or other substance; it appears to be genetically determined.
 Option C: Synergism, is a drug interaction in which the sum of the drug’s
combined effects is greater than that of their separate effects.

10. Answer: D. Order a hemoglobin and hematocrit count 1 hour after the
arteriography

 Option D: A hemoglobin and hematocrit count would be ordered by the


physician if bleeding were suspected.
 Options A, B, and C: The other answers are appropriate nursing interventions
for a patient who has undergone femoral arteriography.

11. Answer: A. Is a protective response to clear the respiratory tract of irritants

 Option A: Coughing, a protective response that clears the respiratory tract of


irritants, usually is involuntary.
 Option B: However, it can be voluntary as when a patient is taught to perform
coughing exercises.
 Option C: An antitussive drug inhibits coughing.
 Option D: Splinting the abdomen supports the abdominal muscles when a
patient coughs.

12. Answer: C. Provide additional bedclothes

 Option C: In an infected patient, shivering results from the body’s attempt to


increase heat production and the production of neutrophils and phagocytic
action through increased skeletal muscle tension and contractions. Initial
vasoconstriction may cause skin to feel cold to the touch. Applying additional
bed clothes helps to equalize the body temperature and stop the chills. Attempts
to cool the body result in further shivering, increased metabolism, and thus
increased heat production.

13. Answer: D. Completed a master’s degree in the prescribed clinical area and is a
registered professional nurse.

 Option D: A clinical nurse specialist must have completed a master’s degree in


a clinical specialty and be a registered professional nurse.
 Option A: The National League of Nursing accredits educational programs in
nursing and provides a testing service to evaluate student nursing competence
but it does not certify nurses.
 Option B: The American Nurses Association identifies requirements for
certification and offers examinations for certification in many areas of nursing,
such as medical-surgical nursing. This certification (credentialing)
demonstrates that the nurse has the knowledge and the ability to provide high-
quality nursing care in the area of her certification.
 Option C: A graduate of an associate degree program is not a clinical nurse
specialist: however, she is prepared to provide bedside nursing with a high
degree of knowledge and skill. She must successfully complete the licensing
examination to become a registered professional nurse.

14. Answer: D. Inhibit the growth of microorganisms

 Option D: Microorganisms usually do not grow in an acidic environment.

15. Answer: D. Bile obstruction

 Option D: Bile colors the stool brown. Any inflammation or obstruction that
impairs bile flow will affect the stool pigment, yielding light, clay-colored
stool.
 Option A: Upper GI bleeding results in black or tarry stool.
 Option B: Constipation is characterized by small, hard masses.
 Option C: Many medications and foods will discolor stool – for example,
drugs containing iron turn stool black.; beets turn stool red.

16. Answer: D. Evaluation

 Option D: In the evaluation step of the nursing process, the nurse must decide
whether the patient has achieved the expected outcome that was identified in
the planning phase.

17. Answer: A. White potatoes

ADVERTISEMENTS

 Option A: The main sources of vitamin A are yellow and green vegetables
(such as carrots, sweet potatoes, squash, spinach, collard greens, broccoli, and
cabbage) and yellow fruits (such as apricots, and cantaloupe). Animal sources
include liver, kidneys, cream, butter, and egg yolks.

18. Answer: D. Maintain the drainage tubing and collection bag below bladder level
to facilitate drainage by gravity

 Option D: Maintaining the drainage tubing and collection bag level with the
patient’s bladder could result in reflux of urine into the kidney. Irrigating the
bladder with Neosporin and clamping the catheter for 1 hour every 4 hours
must be prescribed by a physician.

19. Answer: D. All of the above

 Option D: The ELISA test of venous blood is used to assess blood and
potential blood donors to human immunodeficiency virus (HIV). A positive
ELISA test combined with various signs and symptoms helps to diagnose
acquired immunodeficiency syndrome (AIDS)

20. Answer: A. Subclavian and jugular veins


 Option A: Total Parenteral Nutrition (TPN) requires the use of a large vessel,
such as the subclavian or jugular vein, to ensure rapid dilution of the solution
and thereby prevent complications, such as hyperglycemia. The brachial and
femoral veins usually are contraindicated because they pose an increased risk
of thrombophlebitis.

21. Answer: D. Having the patient shower with an antiseptic soap on the evening
before and the morning of surgery

 Option D: Studies have shown that showering with an antiseptic soap before
surgery is the most effective method of removing microorganisms from the
skin.
 Option A: Shaving the site of the intended surgery might cause breaks in the
skin, thereby increasing the risk of infection; however, if indicated, shaving,
should be done immediately before surgery, not the day before.
 Option B: A topical antiseptic would not remove microorganisms and would
be beneficial only after proper cleaning and rinsing.
 Option C: Tub bathing might transfer organisms to another body site rather
than rinse them away.

22. Answer: C. Leg muscles

 Option C: The leg muscles are the strongest muscles in the body and should
bear the greatest stress when lifting. Muscles of the abdomen, back, and upper
arms may be easily injured.

23. Answer: C. An impaired or traumatized blood vessel wall

 Option C: The factors, known as Virchow’s triad, collectively predispose a


patient to thrombophlebitis; impaired venous return to the heart, blood
hypercoagulability, and injury to a blood vessel wall.
 Option A: Increased partial thromboplastin time indicates a prolonged
bleeding time during fibrin clot formation, commonly the result of
anticoagulant (heparin) therapy.
 Options B and D: Arterial blood disorders (such as pulsus paradoxus) and lung
diseases (such as COPD) do not necessarily impede venous return of injured
vessel walls.

24. Answer: A. Respiratory acidosis, atelectasis, and hypostatic pneumonia

 Option A: Because of restricted respiratory movement, a recumbent,


immobilize patient is at particular risk for respiratory acidosis from poor gas
exchange; atelectasis from reduced surfactant and accumulated mucus in
the bronchioles, and hypostatic pneumonia from bacterial growth caused by
stasis of mucus secretions.

25. Answer: B. Urine retention, bladder distention, and infection

 Option B: The immobilized patient commonly suffers from urine retention


caused by decreased muscle tone in the perineum. This leads to bladder
distention and urine stagnation, which provide an excellent medium for
bacterial growth leading to infection.
 Options A, C, and D: Immobility also results in more alkaline urine with
excessive amounts of calcium, sodium and phosphate, a gradual decrease in
urine production, and an increased specific gravity.
Health assessment

1. Mr. Teban is a 73-year old patient diagnosed with pneumonia. Which data would
be of greatest concern to the nurse when completing the nursing assessment of the
patient?

A. Alert and oriented to date, time, and place


B. Buccal cyanosis and capillary refill greater than 3 seconds
C. Clear breath sounds and nonproductive cough
D. Hemoglobin concentration of 13 g/dl and leukocyte count 5,300/mm3

2. During the nursing assessment, which data represent information concerning


health beliefs?

A. Family role and relationship patterns


B. Educational level and financial status
C. Promotive, preventive, and restorative health practices
D. Use of prescribed and over-the-counter medications

3. Nurse Patrick is acquiring information from a client in the emergency


department. Which is an example of biographic information that may be obtained
during a health history?

A. The chief complaint


B. Past health status
C. History immunizations
D. Location of an advance directive

4. John Joseph was scheduled for a physical assessment. When percussing the
client’s chest, the nurse would expect to find which assessment data as a normal sign
over his lungs?
A. Dullness
B. Resonance
C. Hyperresonance
D. Tympany

5. Matteo is diagnosed with dehydration and underwent series of tests. Which


laboratory result would warrant immediate intervention by the nurse?

A. Serum sodium level of 138 mEq/L


B. Serum potassium level of 3.1 mEq/L
C. Serum glucose level of 120 mg/dl
D. Serum creatinine level of 0.6 mg/100 ml

6. During an otoscopic examination, which action should be avoided to prevent the


client from discomfort and injury?

A. Tipping the client’s head away from the examiner and pulling the ear up and back
B. Inserting the otoscope inferiorly into the distal portion of the external canal
C. Inserting the otoscope superiorly into the proximal two-thirds of the external canal
D. Bracing the examiner’s hand against the client’s head

7. When assessing the lower extremities for arterial function, which intervention
should the nurse perform?

A. Assessing the medial malleoli for pitting edema


B. Performing Allen’s test
C. Assessing the Homans’ sign
D. Palpating the pedal pulses

8. Newly hired nurse Liza is excited to perform her very first physical assessment
with a 19-year-old client. Which assessment examination requires Liza to wear
gloves?
A. Breast
B. Integumentary
C. Ophthalmic
D. Oral

9. Nurse Renor is about to perform Romberg’s test to Pierro. To ensure the latter’s
safety, which intervention should nurse Renor implement?

A. Allowing the client to keep his eyes open


B. Having the client hold on to furniture
C. Letting the client spread his feet apart
D. Standing close to provide support

10. Physical assessment is being performed to Geoff by Nurse Tine. During the
abdominal examination, Tine should perform the four physical examination
techniques in which sequence?

A. Auscultation immediately after inspection and then percussion and palpation


B. Percussion, followed by inspection, auscultation, and palpation
C. Palpation of tender areas first and then inspection, percussion, and auscultation
D. Inspection and then palpation, percussion, and auscultation

11. Which assessment data should the nurse include when obtaining a review of
body systems

A. Brief statement about what brought the client to the health care provider
B. Client complaints of chest pain, dyspnea, or abdominal pain
C. Information about the client’s sexual performance and preference
D. The client’s name, address, age, and phone number

12. Tywin has come to the nursing clinic for a comprehensive health assessment.
Which statement would be the best way to end the history interview?
A. “What brought you to the clinic today?”
B. “Would you describe your overall health as good?”
C. “Do you understand what is happening?”
D. “Is there anything else you would like to tell me?”

13. For which time period would the nurse notify the health care provider that the
client had no bowel sounds?

A. 2 minutes
B. 3 minutes
C. 4 minutes
D. 5 minutes

14. Evaluating the apical pulse is the most reliable noninvasive way to assess cardiac
function. Which is the best area for auscultating the apical pulse?

A. Aortic arch
B. Pulmonic area
C. Tricuspid area
D. Mitral area

15. Beginning in their 20s, women should be told about the benefits and limitations
of breast self-exam (BSE). Which scientific rationale should the nurse remember
when performing a breast examination on a female client?

A. One half of all breast cancer deaths occur in women ages 35 to 45


B. The tail of Spence area must be included in self-examination
C. The position of choice for the breast examination is supine
D. A pad should be placed under the opposite scapula of the breast being palpated

16. Mr. Lim, who has chronic pain, loss of self-esteem, no job, and bodily
disfigurement from severe burns over the trunk and arms, is admitted to a pain
center. Which evaluation criteria would indicate the client’s successful
rehabilitation?

A. The client remains free of the aftermath phase of the pain experience.
B. The client experiences decreased frequency of acute pain episodes.
C. The client continues normal growth and development with intact support systems.
D. The client develops increased tolerance for severe pain in the future.

17. Christine Ann is about to take her NCLEX examination next week and is
currently reviewing the concept of pain. Which scientific rationale would indicate
that she understands the topic?

A. Pain is an objective sign of a more serious problem


B. Pain sensation is affected by a client’s anticipation of pain
C. Intractable pain may be relieved by treatment
D. Psychological factors rarely contribute to a client’s pain perception

18. Miggy, a 6-year-old boy, received a small paper cut on his finger, his mother let
him wash it and apply small amount of antibacterial ointment and bandage. Then
she let him watch TV and eat an apple. This is an example of which type of pain
intervention?

A. Pharmacologic therapy
B. Environmental alteration
C. Control and distraction
D. Cutaneous stimulation

19. Which statement represents the best rationale for using noninvasive and non-
pharmacologic pain-control measures in conjunction with other measures?

A. These measures are more effective than analgesics.


B. These measures decrease input to large fibers.
C. These measures potentiate the effects of analgesics.
D. These measures block transmission of type C fiber impulses.

20. When evaluating a client’s adaptation to pain, which behavior indicates


appropriate adaptation?

A. The client distracts himself during pain episodes.


B. The client denies the existence of any pain.
C. The client reports no need for family support.
D. The client reports pain reduction with decreased activity.

21. In planning pain reduction interventions, which pain theory provides


information most useful to nurses?

A. Specificity theory
B. Pattern theory
C. Gate-control theory
D. Central-control theory

22. Ryan underwent an open reduction and internal fixation of the left hip. One day
after the operation, the client is complaining of pain. Which data would cause the
nurse to refrain from administering the pain medication and to notify the health
care provider instead?

A. Left hip dressing dry and intact


B. Blood pressure of 114/78 mm Hg; pulse rate of 82 beats per minute
C. Left leg in functional anatomic position
D. Left foot cold to touch; no palpable pedal pulse

23. Which term would the nurse use to document pain at one site that is perceived in
other site?
A. Referred pain
B. Phantom pain
C. Intractable pain
D. Aftermath of pain

24. Chuck, who is in the hospital, complains of abdominal pain that ranks 9 on a
scale of 1 (no pain) to 10 (worst pain). Which interventions should the nurse
implement? (Select all that apply.)

A. Assessing the client’s bowel sounds


B. Taking the client’s blood pressure and apical pulse
C. Obtaining a pulse oximeter reading
D. Notifying the health care provider
E. Determining the last time the client received pain medication
F. Encouraging the client to turn, cough, and deep breathe

25. Albert who suffered severe burns 6 months ago is expressing concern about the
possible loss of job-performance abilities and physical disfigurement. Which
intervention is the most appropriate for him?

A. Referring the client for counseling and occupational therapy


B. Staying with the client as much as possible and building trust
C. Providing cutaneous stimulation and pharmacologic therapy
D. Providing distraction and guided imagery techniques

26. Mrs. Bagapayo who had abdominal surgery 3 days earlier complains of sharp,
throbbing abdominal pain that ranks 8 on a scale of 1 (no pain) to 10 (worst pain).
Which intervention should the nurse implement first?

A. Assessing the client to rule out possible complications secondary to surgery


B. Checking the client’s chart to determine when pain medication was last administered
C. Explaining to the client that the pain should not be this severe 3 days postoperatively
D. Obtaining an order for a stronger pain medication because the client’s pain has
increased

27. Which term refers to the pain that has a slower onset, is diffuse, radiates, and is
marked by somatic pain from organs in any body activity?

A. Acute pain
B. Chronic pain
C. Superficial pain
D. Deep pain

28. A 50-year-old widower has arthritis and remains in bed too long because it hurts
to get started. Which intervention should the nurse plan?

A. Telling the client to strictly limit the amount of movement of his inflamed joints
B. Teaching the client’s family how to transfer the client into a wheelchair
C. Teaching the client the proper method for massaging inflamed, sore joints
D. Encouraging gentle range-of-motion exercises after administering aspirin and before
rising

29. Which intervention should the nurse include as a nonpharmacologic pain-relief


intervention for chronic pain?

A. Referring the client for hypnosis


B. Administering pain medication as prescribed
C. Removing all glaring lights and excessive noise
D. Using transcutaneous electric nerve stimulation

ADVERTISEMENTS

30. A 12-year-old student fall off the stairs, grabs his wrist, and cries, “Oh, my
wrist! Help! The pain is so sharp, I think I broke it.” Based on this data, the pain the
student is experiencing is caused by impulses traveling from receptors to the spinal
cord along which type of nerve fibers?

A. Type A-delta fibers


B. Autonomic nerve fibers
C. Type C fibers
D. Somatic efferent fibers

Answers and Rationale

Here are the answers for this exam. Gauge your performance by counter checking your
answers to those below. If you have any disputes or clarifications, please direct them to
the comments section.

1. Answer: B. Buccal cyanosis and capillary refill greater than 3 seconds

Buccal cyanosis and capillary refill greater than 3 seconds are indicative of decreased
oxygen to the tissues, which requires immediate intervention. Alert and oriented, clear
breath sounds, nonproductive cough, hemoglobin concentration of 13 g/dl, and leukocyte
count of 5,300/mm3 are normal data.

2. Answer: C. Promotive, preventive, and restorative health practices

The health-beliefs assessment includes expectations of health care; promotive,


preventive, and restorative practices, such as breast self-examination, testicular
examination, and seat-belt use; and how the client perceives illness. Use of medications
provides information about the client’s personal habits. Educational level, financial
status, and family role and relationship patterns represent information associated with
role and relationship patterns.

3. Answer: D. Location of an advance directive


Biographic information may include name, address, gender, race, occupation, and
location of a living will or a durable power of attorney for health care. The chief
complaint, past health status, and history of immunizations are part of assessing the
client’s health and illness patterns.

4. Answer: B. Resonance

Normally, when percussing a client’s chest, percussion over the lungs reveals resonance,
a hollow or loud, low-pitched sound of long duration. Tympany is typically heard on
percussion over such areas as a gastric air bubble or the intestine. Dullness is typically
heard on percussion of solid organs, such as the liver or areas of consolidation.
Hyperresonance would be evidenced by percussion over areas of overinflation such as an
emphysematous lungs.

5. Answer: B. Serum potassium level of 3.1 mEq/L

A normal potassium level is 3.5 to 5.5 mEq/L. A normal sodium level is 135 to 145
mEq/L, a normal nonfasting glucose level is 85 to 140 mg/dl, and a normal creatinine
level is 0.2 to 0.8 mg/100 ml.

6. Answer: C. Inserting the otoscope superiorly into the proximal two-thirds of the
external canal

In the superior position, the speculum of the otoscope is nearest the tympanic membrane,
and the most sensitive portion of the external canal is the proximal two-thirds. It is
important to avoid these structures during the examination. Tipping the client’s head
away from the examiner, pulling the ear up and back, inserting the otoscope inferiorly,
and bracing the examiner’s hand against the client’s head are all appropriate techniques
used during an otoscopic examination.

7. Answer: D. Palpating the pedal pulses


Palpating the client’s pedal pulses assists in determining if arterial blood supply to the
lower extremities is sufficient. Assessing the medial malleoli for pitting edema is
appropriate for assessing venous function of the lower extremity. Allen’s test is used to
evaluate arterial blood flow before inserting an arterial line in an upper extremity or
obtaining arterial blood gases. Homans’ sign is used to evaluate the possibility of deep
vein thrombosis.

8. Answer: D. Oral

Gloves should be worn any time there is a risk of exposure to the client’s blood or body
fluids. Oral, rectal, and genital examinations require gloves because they involve contact
with body fluids. Ophthalmic, breast, or integumentary examinations normally do not
involve contact with the client’s body fluids and do not require the nurse to wear gloves
for protection. However, if there are areas of skin breakdown or drainage, gloves should
be used.

9. Answer: D. Standing close to provide support

During Romberg’s test, the client is asked to stand with feet together and eyes shut and
still maintain balance with the minimum of sway. If the client loses his balance, the nurse
standing close to provide support, such as having an arm close around his shoulder, can
prevent a fall. Allowing the client to keep his eyes open, spread his feet apart, or hang on
to a piece of furniture interferes with the proper execution of the test and yields invalid
results.

10. Answer: A. Auscultation immediately after inspection and then percussion and
palpation

With an abdominal assessment, auscultation always is performed before percussion and


palpation because any abdominal manipulation, such as from palpation or percussion, can
alter bowel sounds. Percussion should never precede inspection or auscultation, and any
tender or painful areas should be palpated last.
11. Answer: B. Client complaints of chest pain, dyspnea, or abdominal pain

Client complaints about chest pain, dyspnea, or abdominal pain are considered part of the
review of body systems. This potion of the assessment elicits subjective information on
the client’s perceptions of major body system functions, including cardiac, respiratory,
and abdominal. The client’s name, address, age, and phone number are biographical data.
A brief statement about what brought the client to the health care provider is the chief
complaint. Information about the client’s sexual performance and preference addresses
past health status.

12. Answer: D. “Is there anything else you would like to tell me?”

By asking the client if there is anything else, the nurse allows the client to end the
interview by discussing feelings and concerns. Asking about what brought the client to
the clinic is an ambiguous question to which the client may answer “my car” or any
similarly disingenuous reply. Asking if the client describes his overall health as good is a
leading question that puts words in his mouth. Asking if the client understands what is
happening is a yes-or-no question that can elicit little information.

13. Answer: D. 5 minutes

To completely determine that bowel sounds are absent, the nurse must auscultate each of
the four quadrants for at least 5 minutes; 2, 3, or 4 minutes is too short a period to arrive
at this conclusion.

14. Answer: D. Mitral area

The mitral area (also known as the left ventricular area or the apical area), the fifth
intercostal space (ICS) at the left midclavicular line, is the best area for auscultating the
apical pulse. The aortic arch is the second ICS to the right of sternum. The pulmonic area
is the second intercostal space to the left of the sternum. The tricuspid area is the fifth
ICS to the left of the sternum.
15. Answer: B. The tail of Spence area must be included in self-examination

The tail of Spence, an extension of the upper outer quadrant of breast tissue, can develop
breast tumors. This area must also be included in breast self-examination. One half of all
women who die of breast cancer are older than age 65. The correct position for breast
self-examination is not limited to the supine position; the sitting position with hands at
sides, above head, and on the hips is also recommended. A pad is placed under the
ipsilateral (e.g., same side) scapula of the breast being palpated.

16. Answer: C. The client continues normal growth and development with intact
support systems.

Even though the client may experience an aftermath phase, progress is still possible, as is
effective rehabilitation. Aftermath reactions may occur but need not interfere with
rehabilitation. Acute pain is not expected at this stage of recovery. Conditioning probably
would produce less pain tolerance.

17. Answer: B. Pain sensation is affected by a client’s anticipation of pain

Phases of pain experience include the anticipation of pain. Fear and anxiety affect a
person’s response to sensation and typically intensify the pain. Intractable pain is
moderate to severe pain that cannot be relieved by any known treatment. Pain is a
subjective sensation that cannot be quantified by anyone except the person experiencing
it. Psychological factors contribute to a client’s pain perception. In many cases, pain
results from emotions, such as hostility, guilt, or depression.

18. Answer: C. Control and distraction

The mothers actions are example of control and distraction. Involving the child in care
and providing distraction took his mind off the pain. Pharmacologic agents for pain
analgesics — were not used. The home environment was not changed, and cutaneous
stimulation, such as massage, vibration, or pressure, was not used.
19. Answer: C. These measures potentiate the effects of analgesics.

Noninvasive measures may result in release of endogenous molecular neuropeptides with


analgesics properties. They potentiate the effect of analgesics. No evidence indicates that
noninvasive and nonpharmacologic measures are more effective than analgesics in
relieving pain. Decreased input over large fibers allows more pain impulses to reach the
central nervous system. There is no connection between type C fiber impulses and
noninvasive and nonpharmacologic pain-control measures.

20. Answer: A. The client distracts himself during pain episodes.

Distraction is an appropriate method of reducing pain. Denying the existence of any pain
is inappropriate and not indicative of coping. Exclusion of family members and other
sources of support represents a maladaptive response. Range-of-motion exercises and at
least mild activity, not decreased activity, can help reduce pain and are important to
prevent complications of immobility.

21. Answer: D. Central-control theory

No one theory explains all the factors underlying the pain experience, but the central-
control theory discusses brain opiates with analgesic properties and how their release can
be affected by actions initiated by the client and caregivers. The gate-control, specificity,
and patter theories do not address pain control to the depth included in the central-control
theory.

22. Answer: D. Left foot cold to touch; no palpable pedal pulse

A left foot cold to touch without palpable pedal pulse represents an abnormal finding on
neurovascular assessment of the left leg. The client is most likely experiencing some
complication from surgery, which requires immediate medical intervention. The nurse
should notify the health care provider of these findings. A dry and intact hip dressing,
blood pressure of 114/78 mm Hg, pulse rate of 82 beats per minute, and a left foot in
functional anatomic position are all normal assessment findings that do not require
medical intervention.

ADVERTISEMENTS

23. Answer: A. Referred pain

Referred pain is pain occurring at one site that is perceived in another site. Referred pain
follows dermatome and nerve root patterns. Phantom pain refers to pain in a part of the
body that is no longer there, such as in amputation. Intractable pain refers to moderate to
severe pain that cannot be relieved by any known treatment. Aftermath of pain, a phase
of the pain experience and the most neglected phase, addresses the client’s response to
the pain experience.

24. Answer: A, B, and E

The nurse must rule out complications prior to administering pain medication, so her
interventions would include assessing to make sure the client has bowel sounds and
determining if the client is hemorrhaging by checking the client’s blood pressure and
pulse. The nurse must also make sure the pain medication is due according to the health
care provider’s orders. Obtaining a pulse oximeter reading and turning, coughing, and
deep breathing will not help the client’s pain. There is no need to notify the health care
provider in this situation.

25. Answer: A. Referring the client for counseling and occupational therapy

Because it has been 6 months, the client needs professional help to get on with life and
handle the limitations imposed by the current problems. Staying with the client, building
trust, and providing method of pain relief, such as cutaneous stimulation, medications,
distraction, and guided imagery interventions, would have been more appropriate in
earlier stages of postburn injury, when physical pain was most severe and fewer
psychologic factors needed to be addressed.
26. Answer: A. Assessing the client to rule out possible complications secondary to
surgery

The nurse immediate action should be assess the client in an attempt to exclude possible
complications that may be causing the client’s complaints. The health care provider
ordered the pain medication for routine postoperative pain that is expected after
abdominal surgery, not for such complications as hemorrhage, infection, or dehiscence.
The nurse should never administer pain medication without assessing the client first.
Obtaining an order for a strong medication may be appropriate after the nurse assesses
the client and checks the chart to see whether the current analgesic is infective. Checking
the client’s chart is appropriate after the nurse determines that the client is not
experiencing complications from surgery. Pain is subjective, and each person has his own
level of pain tolerance. The nurse must always believe the client’s complaint of pain.

27. Answer: D. Deep pain

Deep pain has a slow onset, is diffuse, and radiates, and is marked by somatic pain from
organs in any body activity. Acute pain is rapid in onset, usually temporary (less than 6
months), and subsides spontaneously. Chronic pain is marked by gradual onset and
lengthy duration (more than 6 months). Superficial pain has abrupt onset with sharp,
stinging quality.

28. Answer: D. Encouraging gentle range-of-motion exercises after administering


aspirin and before rising

Aspirin raises the pain threshold and, although range-of-motion exercises hurt, mild
exercise can relieve pain on rising. Strict limitation of motion only increases the client’s
pain. Having others transfer the client into a wheelchair does not increase his feelings of
dependency. Massage increases inflammation and should be avoided with this client.

29. Answer: D. Using transcutaneous electric nerve stimulation


Nonpharmacologic pain relief interventions include cutaneous stimulation, back rubs,
biofeedback, acupuncture, transcutaneous electric nerve stimulation, and more. Hypnosis
is considered an alternative therapy. Medications are pharmacologic measures. Although
removing glaring lights and excessive noise help to reduce or remove noxious stimuli, it
is not specific to pain relief.

30. Answer: A. Type A-delta fibers

Type A-delta fibers conduct impulses at a very rapid rate and are responsible for
transmitting acute sharp pain signals from the peripheral nerves to the spinal cord. Only
type A-delta fibers transmit sharp, piercing pain. Somatic efferent fibers affect the
voluntary movement of skeletal muscles and joints. Type C fibers transmit sensory input
at a much slower rate and produce a slow, chronic type of pain. The autonomic system
regulates involuntary vital functions and organ control such as breathing.

1. What equipment would be necessary to complete an evaluation of cranial nerves 9


and 10 during a physical assessment?

A. A cotton ball
B. A penlight
C. An ophthalmoscope
D. A tongue depressor and flashlight

2. Which technique would be best in caring for a client following receiving a


diagnosis of a state IV tumor in the brain?

A. Offering the client pamphlets on support groups for brain cancer


B. Asking the client if there is anything he or his family needs
C. Reminding the client that advances in technology are occurring everyday
D. Providing accurate information about the disease and treatment options
3. An 8.5 lb, 6 oz infant is delivered to a diabetic mother. Which nursing
intervention would be implemented when the neonate becomes jittery and lethargic?

A. Administer insulin
B. Administer oxygen
C. Feed the infant glucose water (10%)
D. Place infant in a warmer

4. What question would be most important to ask a male client who is in for a digital
rectal examination?

A. “Have you noticed a change in the force of the urinary system?”


B. “Have you noticed a change in tolerance of certain foods in your diet?”
C. “Do you notice polyuria in the AM?”
D. “Do you notice any burning with urination or any odor to the urine?”

5. The nurse assesses a prolonged late deceleration of the fetal heart rate while the
client is receiving oxytocin (Pitocin) IV to stimulate labor. The priority nursing
intervention would be to:

A. Turn off the infusion


B. Turn the client to the left
C. Change the fluid to Ringer’s Lactate
D. Increase mainline IV rate

6. Which nursing approach would be most appropriate to use while administering


an oral medication to a 4 month old?

A. Place medication in 45cc of formula


B. Place medication in an empty nipple
C. Place medication in a full bottle of formula
D. Place in supine position. Administer medication using a plastic syringe
7. Which nursing intervention would be a priority during the care of a 2-month-old
after surgery?

A. Minimize stimuli for the infant


B. Restrain all extremities
C. Encourage stroking of the infant
D. Demonstrate to the mother how she can assist with her infant’s care.

8. While performing a physical examination on a newborn, which assessment should


be reported to the physician?

A. Head circumference of 40 cm
B. Chest circumference of 32 cm
C. Acrocyanosis and edema of the scalp
D. Heart rate of 160 and respirations of 40

9. Which action by the mother of a preschooler would indicate a disturbed family


interaction?

A. Tells her child that if he does not sit down and shut up she will leave him there.
B. Explains that the injection will burn like a bee sting.
C. Tells her child that the injection can be given while he’s in her lap
D. Reassures child that it is acceptable to cry.

10. During the history, which information from a 21-year-old client would indicate a
risk for development of testicular cancer?

A. Genital Herpes
B. Hydrocele
C. Measles
D. Undescended testicle
11. While caring for a client, the nurse notes a pulsating mass in the client’s
periumbilical area. Which of the following assessments is appropriate for the nurse
to perform?

A. Measure the length of the mass


B. Auscultate the mass
C. Percuss the mass
D. Palpate the mass

12. When observing 4-year-old children playing in the hospital playroom, what
activity would the nurse expect to see the children participating in?

A. Competitive board games with older children


B. Playing with their own toys along side with other children
C. Playing alone with hand held computer games
D. Playing cooperatively with other preschoolers

13. The nurse is teaching the parents of a 3 month-old infant about nutrition. What
is the main source of fluids for an infant until about 12 months of age?

A. Formula or breastmilk
B. Dilute nonfat dry milk
C. Warmed fruit juice
D. Fluoridated tap water

14. While the nurse is administering medications to a client, the client states “I do
not want to take that medicine today.” Which of the following responses by the
nurse would be best?

A. “That’s OK, its alright to skip your medication now and then.”
B. “I will have to call your doctor and report this.”
C. “Is there a reason why you don’t want to take your medicine?”
D. “Do you understand the consequences of refusing your prescribed treatment?”

15. The nurse is assessing a 4 month-old infant. Which motor skill would the nurse
anticipate finding?

A. Hold a rattle
B. Bang two blocks
C. Drink from a cup
D. Wave “bye-bye”

16. The nurse should recognize that all of the following physical changes of the head
and face are associated with the aging client except:

A. pronounced wrinkles on the face.


B. decreased size of the nose and ears.
C. increased growth of facial hair.
D. neck wrinkles.

17. All of the following characteristics would indicate to the nurse that an elder
client might experience undesirable effects of medicines except:

A. increased oxidative enzyme levels.


B. alcohol taken with medication.
C. medications containing magnesium.
D. decreased serum albumin.

18. When assessing a newborn whose mother consumed alcohol during


the pregnancy, the nurse would assess for which of these clinical manifestations?

A. wide-spaced eyes, smooth philtrum, flattened nose


B. strong tongue thrust, short palpebral fissures, simian crease
C. negative Babinski sign, hyperreflexia, deafness
D. shortened limbs, increased jitteriness, constant sucking

19. Which of these statements, when made by the nurse, is most effective when
communicating with a 4-year-old?

A. “Tell me where you hurt.”


B. “Other children like having their blood pressure taken.”
C. “This will be like having a little stick in your arm.”
D. “Anything you tell me is confidential.”

20. A 64-year-old client scheduled for surgery with a general anesthetic refuses to
remove a set of dentures prior to leaving the unit for the operating room. What
would be the most appropriate intervention by the nurse?

A. Explain to the client that the dentures must come out as they may get lost or broken in
the operating room
B. Ask the client if there are second thoughts about having the procedure
C. Notify the anesthesia department and the surgeon of the client’s refusal
D. Ask the client if the preference would be to remove the dentures in the operating room
receiving area

21. The nurse is assessing a client who states her last menstrual period was March
17, and she has missed one period. She reports episodes of nausea and vomiting.
Pregnancy is confirmed by a urine test. What will the nurse calculate as the
estimated date of delivery (EDD)?

A. November 8
B. May 15
C. February 21
D. December 24
22. The family of a 6-year-old with a fractured femur asks the nurse if the child’s
height will be affected by the injury. Which statement is true concerning long bone
fractures in children?

A. Growth problems will occur if the fracture involves the periosteum


B. Epiphyseal fractures often interrupt a child’s normal growth pattern
C. Children usually heal very quickly, so growth problems are rare
D. Adequate blood supply to the bone prevents growth delay after fractures

23. A client is admitted to the hospital with a history of confusion. The client has
difficulty remembering recent events and becomes disoriented when away from
home. Which statement would provide the best reality orientation for this client?

A. “Good morning. Do you remember where you are?”


B. “Hello. My name is Elaine Jones and I am your nurse for today.”
C. “How are you today? Remember, you’re in the hospital.”
D. “Good morning. You’re in the hospital. I am your nurse Elaine Jones.”

24. When a client wishes to improve the appearance of their eyes by removing excess
skin from the face and neck, the nurse should provide teaching regarding which of
the following procedures?

A. Dermabrasion
B. Rhinoplasty
C. Blepharoplasty
D. Rhytidectomy

25. A woman who is six months pregnant is seen in antepartal clinic. She states she
is having trouble with constipation. To minimize this condition, the nurse should
instruct her to
A. increase her fluid intake to three liters/day.
B. request a prescription for a laxative from her physician.
C. stop taking iron supplements.
D. take two tablespoons of mineral oil daily.

Answers and Rationale

1. Answer: D. A tongue depressor and flashlight

ADVERTISEMENTS

Cranial nerves 9 and 10 are the glossopharyngeal and vagus nerves. The gag reflex would
be evaluated.

2. Answer: D. Providing accurate information about the disease and treatment


options

Providing information for the client is the best technique for a new diagnosis.

3. Answer: C. Feed the infant glucose water (10%)

After birth, the infant of a diabetic mother is often hypoglycemic.

4. Answer: A. “Have you noticed a change in the force of the urinary system?”

This change would be most indicative of a potential complication with (BPH) benign
prostate hypertrophy.

5. Answer: A. Turn off the infusion

Stopping the infusion will decrease contractions and possibly remove uterine pressure on
the fetus, which is a possible cause of the deceleration.
6. Answer: B. Place medication in an empty nipple

This is a convenient method for administering medications to an infant. Option D is


partially correct however, the infant is never placed in a reclining position during a
procedure due to a potential aspiration.

7. Answer: C. Encourage stroking of the infant

Tactile stimulation is imperative for an infant’s normal emotional development. After the
trauma of surgery, sensory deprivation can cause failure to thrive.

8. Answer: A. Head circumference of 40 cm

Average circumference of the head for a neonate ranges between 32 to 36 cm. An


increase in size may indicate hydrocephalus or increased intracranial pressure.

9. Answer: A. Tells her child that if he does not sit down and shut up she will leave
him there.

Threatening a child with abandonment will destroy the child’s trust in his family.

10. Answer: D. Undescended testicle

Undescended testicles make the client high risk for testicular cancer. Mumps,
inguinal hernia in childhood, orchitis, and testicular cancer in the contralateral testis are
other predisposing factors.

11. Answer: B. Auscultate the mass

Auscultate the mass. Auscultation of the abdomen and finding a bruit will confirm the
presence of an abdominal aneurysm and will form the basis of information given to the
provider. The mass should not be palpated because of the risk of rupture.
12. Answer: D. Playing cooperatively with other preschoolers

Playing cooperatively with other preschoolers. Cooperative play is typical of the late
preschool period.

13. Answer: A. Formula or breastmilk

Formula or breast milk are the perfect food and source of nutrients and liquids up to 1
year of age.

14. Answer: C. “Is there a reason why you don’t want to take your medicine?”

When a new problem is identified, it is important for the nurse to collect accurate
assessment data. This is crucial to ensure that client needs are adequately identified in
order to select the best nursing care approaches. The nurse should try to discover the
reason for the refusal which may be that the client has developed untoward side effects.

15. Answer: A. Hold a rattle

The age at which a baby will develop the skill of grasping a toy with help is 4 to 6
months.

16. Answer: B. decreased size of the nose and ears.

The nose and ears of the aging client actually become longer and broader. The chin line is
also altered. Wrinkles on the face become more pronounced and tend to take on the
general mood of the client over the years. For example laugh or frown wrinkles about the
eyebrows, lips, cheeks, and outer edges of the eye orbit. The change in the androgen-
estrogen ratio causes an increase in growth of facial hair in most older adults. The aging
process shortens the platysma muscle, which contributes to neck wrinkles.

17. Answer: A. increased oxidative enzyme levels.


Oxidative enzyme levels decrease in the elderly, which affects the disposition of
medication and can alter the therapeutic effects of medication. Alcohol has a smaller
water distribution level in the elderly, resulting in higher blood alcohol levels. Alcohol
also interacts with various drugs to either potentiate or interfere with their effects.
Magnesium is contained in a lot of medications elder clients routinely obtain over the
counter. Magnesium toxicity is a real concern. Albumin is the major drug-binding
protein. Decreased levels of serum albumin mean that higher levels of the drug remain
free and that there are less therapeutic effects and increased drug interactions.

18. Answer: A. wide-spaced eyes, smooth philtrum, flattened nose

The nurse should anticipate that the infant may have fetal alcohol syndrome and should
assess for signs and symptoms of it. These include the characteristics listed in choice A.

19. Answer: A. “Tell me where you hurt.”

Four-year-olds are egocentric and interested in having the focus on themselves. They will
not be interested in what it feels like to other children. Preschoolers are concrete thinkers
and would literally interpret any analogies so they are not helpful in explaining
procedures. Assurance of confidential communication is most appropriate for the
adolescent. In addition, confidentiality is not maintained if the child plans to harm
themselves, harm someone else, or discloses abuse.

20. Answer: D. Ask the client if the preference would be to remove the dentures in
the operating room receiving area

ADVERTISEMENTS

Clients anticipating surgery may experience a variety of fears. This choice allows the
client control over the situation and fosters the client’s sense of self-esteem and self-
concept.

21. Answer: D. December 24


Naegele’s rule: add 7 days and subtract 3 months from the first day of the last regular
menstrual period to calculate the estimated date of delivery.

22. Answer: B. Epiphyseal fractures often interrupt a child’s normal growth pattern

Epiphyseal fractures often interrupt a child’s normal growth pattern

23. Answer is D. “Good morning. You’re in the hospital. I am your nurse Elaine
Jones.”

As cognitive ability declines, the nurse provides a calm, predictable environment for the
client. This response establishes time, location and the caregiver’s name.

24. Answer: D. Rhytidectomy

Rhytidectomy is the procedure for removing excess skin from the face and neck. It is
commonly called a face lift. Dermabrasion involves the spraying of a chemical to cause
light freezing of the skin, which is then abraded with sandpaper or a revolving wire
brush. It is used to remove facial scars, severe acne, and pigment from tattoos.
Rhinoplasty is done to improve the appearance of the nose and involves reshaping the
nasal skeleton and overlying skin. Blepharoplasty is the procedure that removes loose and
protruding fat from the upper and lower eyelids.

25. Answer: A. increase her fluid intake to three liters/day.

In pregnancy, constipation results from decreased gastric motility and increased water
reabsorption in the colon caused by increased levels of progesterone. Increasing fluid
intake to three liters a day will help prevent constipation. The client should increase fluid
intake, increase roughage in the diet, and increase exercise as tolerated. Laxatives are not
recommended because of the possible development of laxative dependence or abdominal
cramping. Iron supplements are necessary during pregnancy, as ordered, and should not
be discontinued. The client should increase fluid intake, increase roughage in the diet,
and increase exercise as tolerated. Laxatives are not recommended because of the
possible development of laxative dependence or abdominal cramping. Mineral oil is
especially bad to use as a laxative because it decreases the absorption of fat-soluble
vitamins (A, D, E, K) if taken near mealtimes.

1. The nurse is caring for an elderly woman who has had a fractured hip repaired.
In the first few days following the surgical repair, which of the following nursing
measures will best facilitate the resumption of activities for this client?

A. arranging for the wheelchair


B. asking her family to visit
C. assisting her to sit out of bed in a chair qid
D. encouraging the use of an overhead trapeze

2. What do you think is the most important nursing order in a client with major
head trauma who is about to receive bolus enteral feeding?

A. measure intake and output.


B. check albumin level.
C. monitor glucose levels.
D. increase enteral feeding.

3. The pathological process causing esophageal varices is:

A. ascites and edema.


B. systemic hypertension.
C. portal hypertension.
D. dilated veins and varicosities.

4. Which of the following interventions will help lessen the effect of GERD (acid
reflux)?
A. Elevate the head of the bed on 4-6 inch blocks.
B. Lie down after eating.
C. Increase fluid intake just before bedtime.
D. Wear a girdle.

5. What is the main benefit of therapeutic massages is:

A. to help a person with swollen legs to decrease the fluid retention.


B. to help a person with duodenal ulcers feel better.
C. to help damaged tissue in a diabetic to heal.
D. to improve circulation and muscles tone.

6. Which of the following foods should be avoided by clients who are prone to
develop heartburn as a result of gastroesophageal reflux disease (GERD)?

A. Lettuce
B. Eggs
C. Chocolate
D. Butterscotch

7. Which of the following should be included in a plan of care for a client


receiving total parenteral nutrition (TPN)?

A. Withhold medications while the TPN is infusing.


B. Change TPN solution every 24 hours.
C. Flush the TPN line with water prior to initiating nutritional support.
D. Keep client on complete bed rest during TPN therapy.

8. Which of the following should be included in a plan of care for a client who is
lactose intolerant?
A. Remove all dairy products from the diet.
B. Frozen yogurt can be included in the diet.
C. Drink small amounts of milk on an empty stomach.
D. Spread out selection of dairy products throughout the day.

9. Pain tolerance in an elderly patient with cancer would:

A. stay the same.


B. be lowered.
C. be increased.
D. no effect on pain tolerance.

10. What is the main advantage of cutaneous stimulation in managing pain:

A. costs less.
B. restricts movement and decreases.
C. gives client control over pain syndrome.
D. allows the family to care for the patient at home.

11. The nurse is instructing a 65-year-old female client diagnosed with osteoporosis.
The most important instruction regarding exercise would be to

A. exercise doing weight bearing activities


B. exercise to reduce weight
C. avoid exercise activities that increase the risk of fracture
D. exercise to strengthen muscles and thereby protect bones

12. A client in a long term care facility complains of pain. The nurse collects data
about the client’s pain. The first step in pain assessment is for the nurse to

A. have the client identify coping methods


B. get the description of the location and intensity of the pain
C. accept the client’s report of pain
D. determine the client’s status of pain

13. Which statement best describes the effects of immobility in children?

A. Immobility prevents the progression of language and fine motor development


B. Immobility in children has similar physical effects to those found in adults
C. Children are more susceptible to the effects of immobility than are adults
D. Children are likely to have prolonged immobility with subsequent complications

14. After a myocardial infarction, a client is placed on a sodium restricted diet.


When the nurse is teaching the client about the diet, which meal plan would be the
most appropriate to suggest?

A. 3 oz. broiled fish, 1 baked potato, ½ cup canned beets, 1 orange, and milk
B. 3 oz. canned salmon, fresh broccoli, 1 biscuit, tea, and 1 apple
C. A bologna sandwich, fresh eggplant, 2 oz fresh fruit, tea, and apple juice
D. 3 oz. turkey, 1 fresh sweet potato, 1/2 cup fresh green beans, milk, and 1 orange

15. A nurse is assessing several clients in a long term health care facility. Which
client is at highest risk for development of decubitus ulcers?

A. A 79 year-old malnourished client on bed rest


B. An obese client who uses a wheelchair
C. An incontinent client who has had 3 diarrhea stools
D. An 80 year-old ambulatory diabetic client

16. Mrs. Kennedy had a CVA (cerebrovascular accident) and has severe right-sided
weakness. She has been taught to walk with a cane. The nurse is evaluating her use
of the cane prior to discharge. Which of the following reflects correct use of the
cane?
A. Holding the cane in her left hand, Mrs. Kennedy moves the cane forward first, then
her right leg, and finally her left leg
B. Holding the cane in her right hand, Mrs. Kennedy moves the cane forward first, then
her left leg, and finally her right leg
C. Holding the cane in her right hand, Mrs. Kennedy moves the cane and her right leg
forward, then moves her left leg forward.
D. Holding the cane in her left hand, Mrs. Kennedy moves the cane and her left leg
forward, then moves her right leg forward

17. The nurse is instructing a woman in a low-fat, high-fiber diet. Which of the
following food choices, if selected by the client, indicate an understanding of a low-
fat, high-fiber diet?

A. Tuna salad sandwich on whole wheat bread.


B. Vegetable soup made with vegetable stock, carrots, celery, and legumes served with
toasted oat bread
C. Chef’s salad with hard boiled eggs and fat-free dressing
D. Broiled chicken stuffed with chopped apples and walnuts

18. An 85-year-old male patient has been bedridden for two weeks. Which of the
following complaints by the patient indicates to the nurse that he is developing a
complication of immobility?

A. Stiffness of the right ankle joint


B. Soreness of the gums
C. Short-term memory loss.
D. Decreased appetite.

19. An eleven-month-old infant is brought to the pediatric clinic. The nurse suspects
that the child has iron deficiency anemia. Because iron deficiency anemia is
suspected, which of the following is the most important information to obtain from
the infant’s parents?
A. Normal dietary intake.
B. Relevant socio cultural, economic, and educational background of the family.
C. Any evidence of blood in the stools
D. A history of maternal anemia during pregnancy

20. A 46-year-old female with chronic constipation is assessed by the nurse for a
bowel training regimen. Which factor indicates further information is needed by the
nurse?

A. The client’s dietary habits include foods high in bulk.


B. The client’s fluid intake is between 2500-3000 ml per day
C. The client engages in moderate exercise each day
D. The client’s bowel habits were not discussed.

ADVERTISEMENTS

Answers and Rationale

1. Answer: D. encouraging the use of an overhead trapeze

Exercise is important to keep the joints and muscles functioning and to prevent secondary
complications. Using the overhead trapeze prevents hazards of immobility by permitting
movement in bed and strengthening of the upper extremities in preparation for
ambulation. Sitting in a wheelchair would require too great hip flexion initially. Asking
her family to visit would not facilitate the resumption of activities. Sitting in a chair
would cause too much hip flexion. The client initially needs to be in a low Fowler’s
position or taking a few steps (as ordered) with the aid of a walker.

2. Answer: A. measure intake and output

It is important to measure intake and output, which should equal. Enteral feeding are
hyperosmotic agents pulling fluid from cells into vascular bed. Water given before
feeding will present a hyperosmotic diuresis. I and O measures assess fluid balance.
3. Answer: C. portal hypertension

Esophageal varices results from increased portal hypertension. In portal hypertension, the
liver cannot accept all of the fluid from the portal vein. The excess fluid will back flow to
the vessels with lesser pressure, such as esophageal veins or rectal veins causing
esophageal varices or hemorrhoids.

4. Answer: A. Elevate the head of the bed on 4-6 inch blocks

Elevation of the head of the bed allows gravity to assist in decreasing the backflow of
acid into the esophagus. Fluid does not flow uphill. The other three options all increase
fluid backflow into the esophagus through position or increasing abdominal pressure.

5. Answer: D. to improve circulation and muscles tone

Particularly in the elderly adults, therapeutic massage will help improve circulation
and muscle tone as well as the personal attention and social interaction that a good
massage provides. A massage is contraindicated in any condition where massage to
damaged tissue can dislodge a blood clot.

6. Answer: C. Chocolate

Ingestion of chocolate can reduce lower esophageal sphincter (LES) pressure leading to
reflux and clinical symptoms of GERD. All of the other foods do not affect LES pressure.

7. Answer: B. Change TPN solution every 24 hours

TPN solutions should be changed every 24 hours in order to prevent bacterial overgrowth
due to hypertonicity of the solution. Option 1 is incorrect; medication therapy can
continue during TPN therapy. Option 3 is incorrect; flushing is not required because the
initiation of TPN does not require a client to remain on bed rest during therapy. However,
other clinical conditions of the client may affect mobility issues and warrant the client’s
being on bed rest.

8. Answer: B. Frozen yogurt can be included in the diet

Clients who are lactose intolerant can digest frozen yogurt. Yogurt products are formed
by bacterial action, and this action assists in the digestion of lactose. The freezing process
further stops bacterial action so that limited lactase activity remains. Option 1 is
incorrect; elimination of all dairy products can lead to significant clinical deficiencies of
other nutrients. Option 3 is incorrect because drinking milk on an empty stomach can
exacerbate clinical symptoms. Drinking milk with a meal may benefit the client because
other foods, (especially fat) may decrease transit time and allow for increased lactase
activity. Option 4 is incorrect because although individual tolerance should be
acknowledged, spreading out the use of known dairy products will usually exacerbate
clinical symptoms.

9. Answer: B. be lowered

There is potential for a lowered pain tolerance to exist with diminished adaptative
capacity.

10. Answer: C. gives client control over pain syndrome.

Cutaneous stimulation allows the patient to have control over his pain and allows him to
be in his own environment. Cutaneous stimulation increases movement and decreases
pain.

11. Answer: A. exercise doing weight bearing activities

Weight bearing exercises are beneficial in the treatment of osteoporosis. Although loss of
bone cannot be substantially reversed, further loss can be greatly reduced if the client
includes weight bearing exercises along with estrogen replacement and calcium
supplements in their treatment protocol.

12. Answer: C. accept the client’s report of pain

Although all of the options above are correct, the first and most important piece of
information in this client’s pain assessment is what the client is telling you about the pain
–“the client’s report.”

13. Answer: B. Immobility in children has similar physical effects to those found in
adults

Care of the immobile child includes efforts to prevent complications of muscle atrophy,
contractures, skin breakdown, decreased metabolism and bone demineralization.
Secondary alterations also occur in the cardiovascular, respiratory and renal systems.
Similar effects and alterations occur in adults.

14. Answer: D. 3 oz. turkey, 1 fresh sweet potato, 1/2 cup fresh green beans, milk,
and 1 orange

Canned fish and vegetables and cured meats are high in sodium. This meal does not
contain any canned fish and/or vegetables or cured meats

15. Answer: A. A 79 year-old malnourished client on bed rest

Weighing significantly less than ideal body weight increases the number and surface area
of bony prominences which are susceptible to pressure ulcers. Thus, malnutrition is a
major risk factor for decubitus, due in part to poor hydration and inadequate protein
intake.

16. Answer: A. Holding the cane in her left hand, Mrs. Kennedy moves the cane
forward first, then her right leg, and finally her left leg
When a person with weakness on one side uses a cane, there should always be two points
of contact with the floor. When Mrs. Kennedy. moves the cane forward, she has both feet
on the floor, providing stability. As she moves the weak leg, the cane and the strong leg
provide support. Finally, the cane, which is even with the weak leg, provides stability
while she moves the strong leg. She should not hold the cane with her weak arm. The use
of the cane requires arm strength to ensure that the cane provides adequate stability when
standing on the weak leg. The cane should be held in the left hand, the hand opposite the
affected leg. If Mrs. Kennedy. moved the cane and her strong foot at the same time, she
would be left standing on her weak leg at one point. This would be unstable at best; at
worst, impossible

ADVERTISEMENTS

17. Answer: B. Vegetable soup made with vegetable stock, carrots, celery, and
legumes served with toasted oat bread

Mayonnaise in tuna salad is high in fat. The whole wheat bread has some fiber. This
choice shows a low-fat soup (which would have been higher in fat if made with chicken
or beef stock) and high-fiber bread and soup contents (both the vegetables and the
legumes). Salad is high in fiber, but hard boiled eggs are high in fat. There is some fiber
in the apples and walnuts. The walnuts are high in fat, as is the chicken.

18. Answer: A. Stiffness of the right ankle joint

Stiffness of a joint may indicate the beginning of a contracture and/or early muscle
atrophy. Soreness of the gums is not related to immobility. Short-term memory loss is not
related to immobility. Decreased appetite is unlikely to be related to immobility.

19. Answer: A. Normal dietary intake.

Iron deficiency anemia occurs commonly in children 6 to 24 months of age. For the first
4 to 5 months of infancy iron stores laid down for the baby during pregnancy are
adequate. When fetal iron stores are depleted, supplemental dietary iron needs to be
supplied to meet the infant’s rapid growth needs. Iron deficiency may occur in the infant
who drinks mostly milk, which contains no iron, and does not receive adequate dietary
iron or supplemental iron. Daily dietary intake is much more related to the diagnosis of
iron deficiency anemia than is sociocultural, economic, and educational background of
the family. Iron deficiency anemia in an infant is very unlikely to be related to
gastrointestinal bleeding. Anemia during pregnancy is unlikely to be the cause of the
infant’s iron deficiency anemia. Fetal iron stores are drawn from the mother even if she is
anemic.

20. Answer: D. The client’s bowel habits were not discussed.

Foods high in bulk are appropriate. Exercise should be a part of a bowel training regimen.
To assess the client for a bowel training program the factors causing the bowel alteration
should be assessed. A routine for bowel elimination should be based on the client’s
previous bowel habits and alterations in bowel habits that have occurred because of
illness or trauma. The client and the family should assist in the planning of the program
which should include foods high in bulk, adequate exercise, and fluid intake of 2500-
3000 ml.

1. The most important nursing intervention to correct skin dryness is:

A. Avoid bathing the patient until the condition is remedied, and notify the physician
B. Ask the physician to refer the patient to a dermatologist, and suggest that the patient
wear home-laundered sleepwear
C. Consult the dietitian about increasing the patient’s fat intake, and take necessary
measures to prevent infection
D. Encourage the patient to increase his fluid intake, use non-irritating soap when bathing
the patient, and apply lotion to the involved areas

2. When bathing a patient’s extremities, the nurse should use long, firm strokes
from the distal to the proximal areas. This technique:
A. Provides an opportunity for skin assessment
B. Avoids undue strain on the nurse
C. Increases venous blood return
D. Causes vasoconstriction and increases circulation

3. Vivid dreaming occurs in which stage of sleep?

A. Stage I non-REM
B. Rapid eye movement (REM) stage
C. Stage II non-REM
D. Delta stage

4. The natural sedative in meat and milk products (especially warm milk) that can
help induce sleep is:

A. Flurazepam
B. Temazepam
C. Tryptophan
D. Methotrimeprazine

5. Nursing interventions that can help the patient to relax and sleep restfully include
all of the following except:

A. Have the patient take a 30- to 60-minute nap in the afternoon


B. Turn on the television in the patient’s room
C. Provide quiet music and interesting reading material
D. Massage the patient’s back with long strokes

6. Restraints can be used for all of the following purposes except to:

A. Prevent a confused patient from removing tubes, such as feeding tubes, I.V. lines, and
urinary catheters
B. Prevent a patient from falling out of bed or a chair
C. Discourage a patient from attempting to ambulate alone when he requires assistance
for his safety
D. Prevent a patient from becoming confused or disoriented

7. Which of the following is the nurse’s legal responsibility when applying


restraints?

A. Document the patient’s behavior


B. Document the type of restraint used
C. Obtain a written order from the physician except in an emergency, when the patient
must be protected from injury to himself or others
D. All of the above

8. Kubler-Ross’s five successive stages of death and dying are:

A. Anger, bargaining, denial, depression, acceptance


B. Denial, anger, depression, bargaining, acceptance
C. Denial, anger, bargaining, depression acceptance
D. Bargaining, denial, anger, depression, acceptance

9. A terminally ill patient usually experiences all of the following feelings during the
anger stage except:

A. Rage
B. Envy
C. Numbness
D. Resentment

10. Nurses and other health care provides often have difficulty helping a terminally
ill patient through the necessary stages leading to acceptance of death. Which of the
following strategies is most helpful to the nurse in achieving this goal?
A. Taking psychology courses related to gerontology
B. Reading books and other literature on the subject of thanatology
C. Reflecting on the significance of death
D. Reviewing varying cultural beliefs and practices related to death

11. Which of the following symptoms is the best indicator of imminent death?

A. A weak, slow pulse


B. Increased muscle tone
C. Fixed, dilated pupils
D. Slow, shallow respirations

12. A nurse caring for a patient with an infectious disease who requires isolation
should refers to guidelines published by the:

A. National League for Nursing (NLN)


B. Centers for Disease Control (CDC)
C. American Medical Association (AMA)
D. American Nurses Association (ANA)

13. To institute appropriate isolation precautions, the nurse must first know the:

A. Organism’s mode of transmission


B. Organism’s Gram-staining characteristics
C. Organism’s susceptibility to antibiotics
D. Patient’s susceptibility to the organism

14. Which is the correct procedure for collecting a sputum specimen for culture and
sensitivity testing?

A. Have the patient place the specimen in a container and enclose the container in a
plastic bag
B. Have the patient expectorate the sputum while the nurse holds the container
C. Have the patient expectorate the sputum into a sterile container
D. Offer the patient an antiseptic mouthwash just before he expectorate the sputum

15. An autoclave is used to sterilize hospital supplies because:

A. More articles can be sterilized at a time


B. Steam causes less damage to the materials
C. A lower temperature can be obtained
D. Pressurized steam penetrates the supplies better

16. The best way to decrease the risk of transferring pathogens to a patient when
removing contaminated gloves is to:

A. Wash the gloves before removing them


B. Gently pull on the fingers of the gloves when removing them
C. Gently pull just below the cuff and invert the gloves when removing them
D. Remove the gloves and then turn them inside out

17. After having an I.V. line in place for 72 hours, a patient complains of tenderness,
burning, and swelling. Assessment of the I.V. site reveals that it is warm and
erythematous. This usually indicates:

A. Infection
B. Infiltration
C. Phlebitis
D. Bleeding

18. To ensure homogenization when diluting powdered medication in a vial, the


nurse should:
A. Shake the vial vigorously
B. Roll the vial gently between the palms
C. Invert the vial and let it stand for 1 minute
D. Do nothing after adding the solution to the vial

19. The nurse is teaching a patient to prepare a syringe with 40 units of U-100
NPH insulin for self-injection. The patient’s first priority concerning self-injection
in this situation is to:

A. Assess the injection site


B. Select the appropriate injection site
C. Check the syringe to verify that the nurse has removed the prescribed insulin dose
D. Clean the injection site in a circular manner with alcohol sponge

20. The physician’s order reads “Administer 1 g cefazolin sodium (Ancef) in 150 ml
of normal saline solution in 60 minutes.” What is the flow rate if the drop factor is
10 gtt = 1 ml?

A. 25 gtt/minute
B. 37 gtt/minute
C. 50 gtt/minute
D. 60 gtt/minute

21. A patient must receive 50 units of Humulin regular insulin. The label reads 100
units = 1 ml. How many milliliters should the nurse administer?

A. 0.5 ml
B. 0.75 ml
C. 1 ml
D. 2 ml
22. How should the nurse prepare an injection for a patient who takes both regular
and NPH insulin?

A. Draw up the NPH insulin, then the regular insulin, in the same syringe
B. Draw up the regular insulin, then the NPH insulin, in the same syringe
C. Use two separate syringe
D. Check with the physician

23. A patient has just received 30 mg of codeine by mouth for pain. Five minutes
later he vomits. What should the nurse do first?

A. Call the physician


B. Remedicate the patient
C. Observe the emesis
D. Explain to the patient that she can do nothing to help him

ADVERTISEMENTS

24. A patient is characterized with a #16 indwelling urinary (Foley) catheter to


determine if:

A. Trauma has occurred


B. His 24-hour output is adequate
C. He has a urinary tract infection
D. Residual urine remains in the bladder after voiding

25. A staff nurse who is promoted to assistant nurse manager may feel
uncomfortable initially when supervising her former peers. She can best decrease
this discomfort by:

A. Writing down all assignments


B. Making changes after evaluating the situation and having discussions with the staff.
C. Telling the staff nurses that she is making changes to benefit their performance
D. Evaluating the clinical performance of each staff nurse in a private conference

Answers and Rationale

1. Answer: D. Encourage the patient to increase his fluid intake, use non-irritating
soap when bathing the patient, and apply lotion to the involved areas

Dry skin will eventually crack, ranking the patient more prone to infection. To prevent
this, the nurse should provide adequate hydration through fluid intake, use nonirritating
soaps or no soap when bathing the patient, and lubricate the patient’s skin with lotion.
Bathing may be limited but need not be avoided entirely. The attending physician and
dietitian may be consulted for treatment, but home-laundered items usually are not
necessary.

2. Answer: C. Increases venous blood return

Washing from distal to proximal areas stimulates venous blood flow, thereby preventing
venous stasis. It improves circulation but does not result in vasoconstriction. The nurse
can assess the patient’s condition throughout the bath, regardless of washing technique,
and should feel no strain while bathing the patient.

3. Answer: B. Rapid eye movement (REM) stage

Other characteristics of rapid eye movement (REM) sleep are deep sleep (the patient
cannot be awakened easily), depressed muscle tone, and possibly irregular heart and
respiratory rates. Non-REM sleep is a deep, restful sleep without dreaming. Delta stage,
or slow-wave sleep, occurs during non-REM Stages III and IV and is often equated with
quiet sleep.

4. Answer: C. Tryptophan
Tryptophan is a natural sedative; flurazepam (Dalmane), temazepam (Restoril), and
methotrimeprazine (Levoprome) are hypnotic sedatives.

5. Answer: A. Have the patient take a 30- to 60-minute nap in the afternoon

Napping in the afternoon is not conductive to nighttime sleeping. Quiet music, watching
television, reading, and massage usually will relax the patient, helping him to fall asleep.

6. Answer: D. Prevent a patient from becoming confused or disoriented

By restricting a patient’s movements, restraints may increase stress and lead to confusion,
rather than prevent it. The other choices are valid reasons for using restraints.

7. Answer: D. All of the above

When applying restraints, the nurse must document the type of behavior that prompted
her to use them, document the type of restraints used, and obtain a physician’s written
order for the restraints.

8. Answer: C. Denial, anger, bargaining, depression acceptance

Kubler-Ross’s five successive stages of death and dying are denial, anger, bargaining,
depression, and acceptance. The patient may move back and forth through the different
stages as he and his family members react to the process of dying, but he usually goes
through all of these stages to reach acceptance.

9. Answer: C. Numbness

Numbness is typical of the depression stage, when the patient feels a great sense of loss.
The anger stage includes such feelings as rage, envy, resentment, and the patient’s
questioning “Why me?”
10. Answer: C. Reflecting on the significance of death

According to thanatologists, reflecting on the significance of death helps to reduce


the fear of death and enables the health care provider to better understand the terminally
ill patient’s feelings. It also helps to overcome the belief that medical and nursing
measures have failed, when a patient cannot be cured.

11. Answer: C. Fixed, dilated pupils

Fixed, dilated pupils are sign of imminent death. Pulse becomes weak but rapid, muscles
become weak and atonic, and periods of apnea occur during respiration.

12. Answer: B. Centers for Disease Control (CDC)

The Center of Disease Control (CDC) publishes and frequently updates guidelines on
caring for patients who require isolation. The National League of Nursing’s (NLN’s)
major function is accrediting nursing education programs in the United States. The
American Medical Association (AMA) is a national organization of physicians. The
American Nurses’ Association (ANA) is a national organization of registered nurses.

13. Answer: A. Organism’s mode of transmission

Before instituting isolation precaution, the nurse must first determine the organism’s
mode of transmission. For example, an organism transmitted through nasal secretions
requires that the patient be kept in respiratory isolation, which involves keeping the
patient in a private room with the door closed and wearing a mask, a gown, and gloves
when coming in direct contact with the patient. The organism’s Gram-straining
characteristics reveal whether the organism is gram-negative or gram-positive, an
important criterion in the physician’s choice for drug therapy and the nurse’s
development of an effective plan of care. The nurse also needs to know whether the
organism is susceptible to antibiotics, but this could take several days to determine; if she
waits for the results before instituting isolation precautions, the organism could be
transmitted in the meantime. The patient’s susceptibility to the organism has already been
established. The nurse would not be instituting isolation precautions for a non-infected
patient.

14. Answer: C. Have the patient expectorate the sputum into a sterile container

Placing the specimen in a sterile container ensures that it will not become contaminated.
The other answers are incorrect because they do not mention sterility and because
antiseptic mouthwash could destroy the organism to be cultured (before sputum
collection, the patient may use only tap water for nursing the mouth).

15. Answer: D. Pressurized steam penetrates the supplies better

An autoclave, an apparatus that sterilizes equipment by means of high-temperature


pressurized steam, is used because it can destroy all forms of microorganisms, including
spores.

16. Answer: C. Gently pull just below the cuff and invert the gloves when removing
them

Turning the gloves inside out while removing them keeps all contaminants inside the
gloves. They should then be placed in a plastic bag with soiled dressings and discarded in
a soiled utility room garbage pail (double bagged). The other choices can spread
pathogens within the environment.

17. Answer: C. Phlebitis

Tenderness, warmth, swelling, and, in some instances, a burning sensation are signs and
symptoms of phlebitis. Infection is less likely because no drainage or fever is present.
Infiltration would result in swelling and pallor, not erythema, near the insertion site. The
patient has no evidence of bleeding.
18. Answer: B. Roll the vial gently between the palms

Gently rolling a sealed vial between the palms produces sufficient heat to enhance
dissolution of a powdered medication. Shaking the vial vigorously can break down the
medication and alter its pharmacologic action. Inverting the vial or leaving it alone does
not ensure thorough homogenization of the powder and the solvent.

ADVERTISEMENTS

19. Answer: C. Check the syringe to verify that the nurse has removed the
prescribed insulin dose

When the nurse teaches the patient to prepare an insulin injection, the patient’s first
priority is to validate the dose accuracy. The next steps are to select the site, assess the
site, and clean the site with alcohol before injecting the insulin.

20. Answer: A. 25 gtt/minute

21. Answer: A. 0.5 ml

22. Answer: B. Draw up the regular insulin, then the NPH insulin, in the same
syringe

Drugs that are compatible may be mixed together in one syringe. In the case of insulin,
the shorter-acting, clear insulin (regular) should be drawn up before the longer-acting,
cloudy insulin (NPH) to ensure accurate measurements.

23. Answer: C. Observe the emesis

After a patient has vomited, the nurse must inspect the emesis to document color,
consistency, and amount. In this situation, the patient recently ingested medication, so the
nurse needs to check for remnants of the medication to help determine whether the
patient retained enough of it to be effective. The nurse must then notify the physician,
who will decide whether to repeat the dose or prescribe an antiemetic.

24. Answer: B. His 24-hour output is adequate

A 24-hour urine output of less than 500 ml in an adult is considered inadequate and may
indicate kidney failure. This must be corrected while the patient is in the acute state so
that appropriate fluids, electrolytes, and medications can be administered and excreted.
Indwelling catheterization is not needed to diagnose trauma, urinary tract infection, or
residual urine.

25. Answer: B. Making changes after evaluating the situation and having
discussions with the staff.

A new assistant nurse manager should not make changes until she has had a chance to
evaluate staff members, patients, and physicians. Changes must be planned thoroughly
and should be based on a need to improve conditions, not just for the sake of change.
Written assignments allow all staff members to know their own and others
responsibilities and serve as a checklist for the manager, enabling her to gauge whether
the unit is being run effectively and whether patients are receiving appropriate care.
Telling the staff nurses that she is making changes to benefit their performance should
occur only after the nurse has made a thorough evaluation. Evaluations are usually done
on a yearly basis or as needed.

Potrebbero piacerti anche